SlideShare a Scribd company logo
1 of 24
Download to read offline
KEMENTERIAN PENDIDIKAN NASIONAL
DIREKTORAT JENDERAL PENDIDIKAN MENENGAH
DIREKTORAT PEMBINAAN SEKOLAH MENENGAH ATAS

TEST TINGKAT KABUPATEN/KOTA
SELEKSI CALON PESERTA
INTERNATIONAL BIOLOGY OLYMPIAD (IBO)
TAHUN 2012
KODE: 03
PETUNJUK:
1. Setiap soal memiliki bobot nilai 1.
2. Isilah nama, asal sekolah, kelas, dan kode soal pada lembar
jawaban.
3. Gunakan ballpoint/pulpen untuk mengisi jawaban yang tepat
pada lembar yang telah disediakan dengan cara mencantumkan
hurufnya saja.
4. Tidak ada sistem minus.
5. Waktu Test = 120 menit.

TIM OLIMPIADE BIOLOGI INDONESIA
2011

03-OSK-2011
Copyright © 2011 by Kemdiknas
Hak Cipta dilindungi oleh Undang-undang

0
Nama: .........................................................................................................
Asal SMA/Kelas: ........................................................................................

Seleksi Calon Peserta IBO 2012
Tingkat Kabupaten/Kota (Kode:03)

BIOLOGI SEL DAN MOLEKULER (Nilai 20)
1.

Manakah pasangan berikut ini yang TIDAK merepresentasikan hubungan yang benar?
A. Lemak; lipid
B. Pati; polisakarida
C. Pati; karbohidrat
D. Gula; karbohidrat
E. Enzim; lipid

2.

Di dalam sel hewan, cadangan makanan berupa lipid disimpan dalam bentuk molekul:
A. asam lemak
B. trigliserida
C. glikogen
D. lipid bilayer
E. badan keton

3.

Perhatikan gambar grafik berikut ini.

Berdasarkan grafik di atas, berapa persentase perubahan pada rasio luas permukaan-volume
apabila sebuah sel berkembang dari diameter 1 m menjadi 2 m?
A. 10%
B. 20%
C. 30%
D. 50%
E. 90%
4.

Berdasarkan grafik pada pertanyaan No.3 di atas, berapakah diameter maksimum yang dapat
ditunjang oleh sel tersebut sebelum rasio luas permukaan - volume mencapai nilai di bawah
1?
A. 2 m
B. 5 m
C. 15 m
D. 10 m
E. 20 m
1
03-OSK-2011
Copyright © 2011 by Kemdiknas
Hak Cipta dilindungi oleh Undang-undang
Nama: .........................................................................................................
Asal SMA/Kelas: ........................................................................................

Seleksi Calon Peserta IBO 2012
Tingkat Kabupaten/Kota (Kode:03)

5.

Sel kelenjar yang mampu memproduksi hormon insulin dalam jumlah besar umumnya
memiliki organel (.....) yang berkembang dengan baik.
A. silia
B. sentriol
C. RE kasar
D. RE halus
E. lisosom

6.

Sel makrofag pada sistem imun berperan dalam menelan bakteri atau zat asing lainnya.
Organel yang berperan penting dalam proses ini adalah:
A. sentriol
B. nukleus
C. peroksisom
D. lisosom
E. RE halus

7.

Menurut anda, apakah yang akan terjadi apabila lisosom pada sel pecah dan mengeluarkan
isinya ke dalam sitoplasma?
A. Peningkatan produksi protein
B. Terjadinya pembelahan mitokondria dan kloroplas
C. Makromolekul di dalam sitosol terdegradasi
D. DNA di dalam mitokondria terdegradasi
E. DNA di dalam kloroplas terdegradasi

8.

Manakah dari pernyataan mengenai struktur dan organel sel berikut yang benar?
A. Pada sel hewan dan tumbuhan, retikulum endoplasma terhubung dengan membran luar
dari nukleus
B. Pada umumnya, nukleus merupakan organel dengan ukuran terbesar pada tumbuhan
multiseluler
C. Sambungan celah (gap junction) dan plasmodesmata pada dasarnya adalah struktur yang
sama, namun diberi nama yang berbeda pada sel hewan dan tumbuhan
D. Sel tumbuhan berbeda dengan sel hewan karena mereka tidak memiliki mitokondria
E. Sel tumbuhan berbeda dengan sel hewan karena mereka tidak memiliki sitoskeleton

9.

Plastida yang berfungsi untuk menyimpan lemak pada sel tumbuhan disebut:
A. kloroplas
B. amiloplas
C. leukoplas
D. kromoplas
E. peroksisom

10. Ketika sel-sel epitel silindris pada saluran pencernaan diberikan suatu senyawa kimia X,
bentuknya berubah menjadi bulat melingkar. Struktur internal sel yang terganggu oleh
senyawa kimia X tersebut adalah:
A. mitokondria
B. membran sel
C. retikulum endoplasma
D. mikrotubul
E. sambungan sel (cell junctions)
2
03-OSK-2011
Copyright © 2011 by Kemdiknas
Hak Cipta dilindungi oleh Undang-undang
Nama: .........................................................................................................
Asal SMA/Kelas: ........................................................................................

Seleksi Calon Peserta IBO 2012
Tingkat Kabupaten/Kota (Kode:03)

11. Dengan memperhatikan peran Ca2+ (kalsium) dalam polimerisasi aktin, maka penurunan
secara signifikan jumlah Ca2+ ekstraseluler pada medium Euglena dapat menyebabkan:
A. sel bergerak cepat
B. sel cenderung membulat dan memendek
C. peningkatan volume sitoplasma
D. pemanjangan membran sel
E. menghambat fluiditas membran
12. Molekul air dapat melewati membran sel melalui:
A. protein perifer
B. aquaporin
C. protein membran dan menggunakan ATP
D. sistem kotransport
E. endositosis
13. Tipe golongan darah A, B, dan O dapat ditandai melalui molekul (.....) pada permukaan sel
A. glikoprotein
B. glikolipid
C. lipoprotein
D. gliserol
E. glutamin
14. Sel-sel di dalam tubuh suatu individu dapat mengenal satu sama lain melalui komponen (.....)
pada membran sel
A. glikolipid
B. posfat
C. glikoprotein
D. lipoprotein
E. ketoglutarat
15. Apa yang akan terjadi jika sel darah merah manusia ditempatkan pada air distilasi?
A. Sel akan mengkerut
B. Sel akan mengalami plasmolisis
C. Sel akan pecah
D. Sel akan kehilangan air
E. Sel tidak mengalami perubahan
16. Threonin (asam amino) dan arabinosa (monosakarida) dapat masuk ke dalam sel melalui:
A. difusi
B. endocytosis
C. fosforilasi
D. difusi terfasilitasi
E. osmosis
17. Proses apakah yang dijelaskan oleh mekanisme kemiosmosis?
A. Pergerakan molekul air ke dalam kloroplas dan penggunaannya dalam fotosintesis
B. Pembentukan ATP yang dipasangkan dengan aliran proton (H+) pada respirasi aerob
C. Pergerakan molekul hidrofobik melintasi membran sel
D. Pergerakan molekul NADH dari sitoplasma ke dalam mitokondria
E. Pembentukan gradien kimiawi oleh osmosis
3
03-OSK-2011
Copyright © 2011 by Kemdiknas
Hak Cipta dilindungi oleh Undang-undang
Nama: .........................................................................................................
Asal SMA/Kelas: ........................................................................................

Seleksi Calon Peserta IBO 2012
Tingkat Kabupaten/Kota (Kode:03)

18. Drosophila melanogaster memiliki empat pasang kromosom. Jika sperma dari spesies ini
terbentuk melalui proses meiosis tanpa mengalami crossing over, berapa banyak kombinasi
genetik pada sel sperma dapat diproduksi oleh jantan Drosophila melanogaster?
A. 4
B. 8
C. 16
D. 64
E. 256
19. Diagram di bawah menunjukkan perubahan jumlah DNA di dalam sel (ordinat/sumbu Y)
sebagai fungsi dari waktu (absis/sumbu X) selama siklus sel. Diagram manakah yang
menunjukkan perubahan jumlah DNA di dalam sel pada waktu meiosis dan mitosis?

A.
B.
C.
D.
E.

Meiosis = 2, Mitosis = 4
Meiosis = 3, Mitosis = 1
Meiosis = 3, Mitosis = 2
Meiosis = 4, Mitosis = 1
Meiosis = 4, Mitosis = 2

20. Salah satu cara penanggulangan influenza adalah melalui vaksinasi, namun hampir setiap
tahun harus dibuat jenis vaksin flu yang baru. Hal ini terutama disebabkan karena:
A. virus yang berbeda menyerang penduduk pada usia yang berbeda sehingga setiap tahun
harus diproduksi vaksin yang baru untuk kelompok usia berbeda
B. antibodi yang diproduksi tubuh terhadap vaksin flu tidak bertahan lama dalam darah
C. vaksin umumnya bersifat tidak stabil sehingga tidak dapat disimpan lebih dari setahun
D. tubuh kita belajar untuk menghancurkan antibodi yang telah diproduksi terhadap vaksin
sehingga diperlukan jenis vaksin baru untuk setiap kali vaksinasi
E. virus influenza memiliki frekuensi mutasi yang tinggi

4
03-OSK-2011
Copyright © 2011 by Kemdiknas
Hak Cipta dilindungi oleh Undang-undang
Nama: .........................................................................................................
Asal SMA/Kelas: ........................................................................................

Seleksi Calon Peserta IBO 2012
Tingkat Kabupaten/Kota (Kode:03)

ANATOMI DAN FISIOLOGI TUMBUHAN (Nilai 15)
1.

Selama fotosintesis, karbohidrat dibuat oleh tumbuhan:
A. selama reaksi terang
B. dari oksigen, air, dan tiga molekul karbondioksida
C. di grana
D. di stroma
E. agar dapat memulai reaksi terang

2.

Urutan yang tepat menggambarkan aliran elektron yang terjadi pada waktu fotosintesis
adalah:
A. air – klorofil – NADP+
B. air – NADP+ – klorofil
C. klorofil – air – NADP+
D. klorofil – NADP+ – air
E. air – NADP+ – klorofil

3.

Jika seorang ilmuwan menambahkan senyawa kimia yang khusus menghambat difusi proton
yang keluar dari tilakoid melalui kompleks ATP sintase, hasil apa yang akan diperoleh?
A. meningkatkan NADP+ pada bagian dalam tilakoid
B. meningkatkan pH di bagian dalam tilakoid
C. produksi ATP yang meningkat
D. tidak ada perubahan dalam produksi ATP
E. produksi ATP menurun

4.

Spektrum penyerapan cahaya pada klorofil a tidak identik dengan spektrum fotosintesis
karena:
A. terdapat pigmen aksesori yang turut berkontribusi pada proses fotosintesis
B. klorofil a menyerap baik cahaya merah maupun biru
C. klorofil a memantulkan cahaya hijau
D. perbedaan panjang gelombang cahaya menghasilkan perbedaan energi yang dihasilkan
E. klorofil a dapat diaktifkan dengan menyerap foton dari cahaya

5.

Fotosintesis terjadi pada:
A. sel-sel sklerenkim
B. sel-sel kolenkim
C. sel-sel penjaga
D. sel-sel parenkim
E. jawaban C dan D

6.

Efek apa yang terjadi pada tumbuhan ketika diberi larutan 10% sukrosa?
A. stomata akan membuka sehingga menyebabkan tumbuhan kehilangan air
B. stomata akan menutup sehingga tumbuhan kekurangan karbondioksida
C. stomata akan menutup sehinggah sehingga tumbuhan kekurangan oksigen
D. sel tumbuhan akan terisi air dari osmosis sehingga tumbuhan menjadi turgid
E. jawaban B dan C

5
03-OSK-2011
Copyright © 2011 by Kemdiknas
Hak Cipta dilindungi oleh Undang-undang
Nama: .........................................................................................................
Asal SMA/Kelas: ........................................................................................

7.

Seleksi Calon Peserta IBO 2012
Tingkat Kabupaten/Kota (Kode:03)

Perhatikan gambar berikut ini.

Tumbuhan yang memiliki anatomi organ seperti gambar di atas memiliki kemampuan untuk:
A. melakukan fotosintesis di mesofil dan seludang pembuluh
B. menghasilkan asam oksalat dari CO2
C. melakukan fotorespirasi
D. jawaban B dan C
E. jawaban A dan B
8.

Tekanan akar pada tumbuhan akan menyebabkan air bergerak pada sel-sel akar melalui
proses:
A. translokasi
B. adhesi
C. osmosis
D. kerja kapiler
E. gutasi

9.

Perhatikan gambar dibawah ini.

Pernyataan yang tepat untuk menjelaskan gambar di atas adalah:
A. nirogen dapat langsung diikat (difiksasi akar tumbuhan)
B. nitrogen dapat diserap langsung oleh stomata
C. nitrogen mengalami perubahan menjadi asam amino oleh aktivitas bakteria
D. nitrogen dapat kembali ditranspirasikan ke atmosfer melalui stomata
E. nitrogen mengalami amonifikasi dengan bantuan bakteri
6
03-OSK-2011
Copyright © 2011 by Kemdiknas
Hak Cipta dilindungi oleh Undang-undang
Nama: .........................................................................................................
Asal SMA/Kelas: ........................................................................................

Seleksi Calon Peserta IBO 2012
Tingkat Kabupaten/Kota (Kode:03)

10. Tumbuhan menjadi kerdil dan mengalami klorosis diakibatkan terjadi defisiensi unsur berikut
secara berturut-turut, yaitu:
A. N-P dan Mg-Ca
B. N-Mg dan Ca-P
C. Mg-P dan N-Ca
D. Mg-Ca dan N-P
E. P-Ca dan Mg-N
11. Selama proses reproduksi pada tumbuhan berbunga, terjadi beberapa perubahan sebagai
berikut, KECUALI:
A. mikrospora berubah menjadi serbuk polen
B. ovarium menjadi buah
C. ovulum menjadi biji
D. kelopak bunga berguguran
E. tabung polen menjadi inti sperma
12. Selama proses reproduksi tumbuhan, semua transisi berikut ini terjadi, KECUALI:
A. mikrospora menjadi butir pollen
B. ovarium menjadi buah
C. ovulum menjadi biji
D. mahkota bunga gugur
E. inti tabung menjadi inti sperma
13. Perhatikan gambar sayatan longitudinal dari ovul tumbuhan angiospermae.

Dengan struktur pada nomor berapakah, isi dari tabung polen akan berfusi?
A. 1 dan 3
B. 1 dan 4
C. 2 dan 3
D. 3 dan 4
E. 2 dan 4
7
03-OSK-2011
Copyright © 2011 by Kemdiknas
Hak Cipta dilindungi oleh Undang-undang
Nama: .........................................................................................................
Asal SMA/Kelas: ........................................................................................

Seleksi Calon Peserta IBO 2012
Tingkat Kabupaten/Kota (Kode:03)

14. Peristiwa berikut yang TIDAK berkaitan dengan peran auksin pada tumbuhan adalah:
A. dominansi apikal
B. partenokarpi
C. mencegah absisi
D. memecah masa istirahat biji (breaking dormancy)
E. memicu pertumbuhan akar adventif
15. Berapa tahun usia batang pohon yang tumbuh di daerah temperata dengan hasil sayatan
melintang seperti gambar di bawah ini?

A.
B.
C.
D.
E.

4 tahun
6 tahun
8 tahun
10 tahun
14 tahun

8
03-OSK-2011
Copyright © 2011 by Kemdiknas
Hak Cipta dilindungi oleh Undang-undang
Nama: .........................................................................................................
Asal SMA/Kelas: ........................................................................................

Seleksi Calon Peserta IBO 2012
Tingkat Kabupaten/Kota (Kode:03)

ANATOMI DAN FISIOLOGI HEWAN (Nilai 25)
1.

Cairan ekstraseluler yang mengelilingi sel-sel penyusun tubuh hewan berasal dari:
A. plasma darah
B. kelenjar eksokrin
C. kelenjar endokrin
D. jaringan limfatik
E. hasil filtrasi ginjal

2.

Manakah material berikut ini yang TIDAK dapat dicerna oleh sapi?
A. Protein
B. Trigliserida
C. Glikogen
D. Selulosa
E. Asam nukleat

3.

Pernyataan di bawah ini yang berlaku untuk seluruh steroid adalah:
A. Seluruh steroid larut di dalam seluruh vitamin
B. Sel target dari setiap steroid adalah setiap sel di dalam tubuh
C. Steroid berikatan kepada reseptor protein yang terdapat pada membran dari sel target
mereka
D. Steroid disintesis pada reticulum endoplasma kasar
E. Steroid larut di dalam senyawa lipid

4.

Pankreas adalah organ tubuh yang unik karena memiliki fungsi eksokrin dan endokrin.
Fungsi eksokrin dari pankreas adalah melepaskan:
A. enzim pencernaan ke darah
B. enzim pencernaan melalui suatu saluran
C. hormon langsung ke darah
D. hormon melalui suatu saluran
E. steroid langsung ke darah

5.

Sistem respiratori penting dalam perolehan oksigen bagi jaringan dan:
A. mengatur pH darah dengan mengendalikan berapa banyak oksigen yang terlarut di dalam
plasma
B. mengatur pH darah dengan mengendalikan berapa karbondioksida yang terlarut di dalam
plasma
C. mengatur tekanan darah dengan melepaskan hormon dari paru-paru yang mengendalikan
laju detak jantung
D. berperan dalam mengatur kadar kolesterol di dalam darah
E. mengambil beberapa nutrien, seperti vitamin K, dari atmosfer saat persediaan terbatas

6.

Manakah dari pernyataan berikut ini yang merupakan karakteristik dari arteri?
A. Mengandung katup yang mencegah aliran balik arah
B. Membawa darah menjauhi jantung
C. Darah dipertahankan bergerak oleh kontraksi otot volunter
D. Merupakan pembuluh yang berdinding tipis
E. Selalu membawa darah yang kaya akan oksigen

9
03-OSK-2011
Copyright © 2011 by Kemdiknas
Hak Cipta dilindungi oleh Undang-undang
Nama: .........................................................................................................
Asal SMA/Kelas: ........................................................................................

Seleksi Calon Peserta IBO 2012
Tingkat Kabupaten/Kota (Kode:03)

7.

Darah mengalir dari jantung ke paru-paru pada arteri pulmonaris dan kembali dari paru-paru
ke jantung pada vena pulmonaris. Darah di arteri pulmonaris:
A. memiliki kandungan O2 lebih tinggi dan kandungan CO2 lebih rendah dari darah yang
berada di vena pulmonaris
B. memiliki kandungan O2 dan CO2 lebih tinggi daripada darah yang berada di vena
pulmonaris
C. memiliki kandungan O2 lebih rendah dan kandungan CO2 lebih tinggi daripada darah
yang berada di vena pulmonaris
D. memiliki kandungan O2 dan CO2 lebih rendah daripada darah yang berada di vena
pulmonaris
E. memiliki kandungan O2 lebih tinggi tetapi kandungan CO2 sama dengan darah yang
berada di vena pulmonaris

8.

Pernyataan berikut yang TIDAK tepat mengenai pundi-pundi udara pada burung adalah:
A. Berperan dalam menampung udara inspirasi atau pun ekspirasi
B. Mensuplai udara kaya O2 ke paru-paru pada saat ekspirasi
C. Bersama-sama dengan paru-paru melakukan pertukaran gas dengan darah
D. Mengalami perubahan ukuran selama proses respirasi berlangsung
E. Menampung udara dari paru-paru saat inspirasi

9.

Tabel berikut adalah data mengenai kecepatan dan tekanan darah selama mengalir di
sepanjang pembuluh darah:

I
II
III
IV
V

Tekanan darah (mm Hg)
100
22
60
2
10

Kecepatan aliran darah (cm/s)
48
0,5
15
25
4

Berdasarkan keterangan diatas maka urutan yang tepat untuk menunjukan perubahan tekanan
dan kecepatan aliran darah selama mengalir dari jantung dan kembali ke jantung adalah:
A. ventrikel-I-III-II-V-IV-atrium
B. ventrikel-IV-V-II-III-I-atrium
C. ventrikel-II-V-III-IV-I-atrium
D. ventrikel-I-IV-III-V-II-atrium
E. ventrikel-I-II-III-IV-V-atrium
10. Dinding arteri lebih tebal dibandingkan dinding vena. Manakah alasan yang tepat untuk
mendukung struktur arteri tersebut?
A. Mengalirkan darah dari jantung
B. Menahan darah untuk kembali
C. Mengandung oksigen dari paru-paru
D. Membawa darah lebih banyak
E. Jawaban A dan B benar

10
03-OSK-2011
Copyright © 2011 by Kemdiknas
Hak Cipta dilindungi oleh Undang-undang
Nama: .........................................................................................................
Asal SMA/Kelas: ........................................................................................

Seleksi Calon Peserta IBO 2012
Tingkat Kabupaten/Kota (Kode:03)

11. Apakah fungsi katup pada vena?
A. Mencegah pembalikan aliran darah
B. Mencegah darah
C. Mencegah terbentuknya plak
D. Memperluas permukaan vena
E. Meningkatkan tekanan darah
12. Pertukaran gas terjadi di arteri sehingga jaringan epitel yang membatasi alveoli memiliki
struktur:
A. selapis kubus
B. selapis pipih
C. tidak ada sel epitel
D. sel endotel
E. silindris berlapis banyak
13. Orang yang memiliki sel darah merah sabit (sickle cell) akan memiliki masalah serius di
dataran tinggi karena:
A. jumlah sel darah merah
B. jumlah hemoglobin di dalam darah
C. konsentrasi oksigen
D. tekanan udara parsial
E. suhu udara
14. Berikut ini adalah molekul-molekul yang terdapat di dalam plasma darah:
I. Urea
II. Air
III. Glukosa
IV. Protein
V. Antibodi
VI. Ion hidrogen
Berapa macam dari molekul di atas yang diekskresikan atau direabsorbsi di ginjal?
A. 3
B. 2
C. 4
D. 5
E. 6
15. Seorang meneliti tiga jenis rodensia yang berkerabat dekat. Yang menjadi objek kajian dalam
penelitian tersebut adalah anatomi perbandingan dari struktur ginjal ketiga jenis rodensia.
Hasil penelitian menunjukan bahwa ketiga jenis rodensia memiliki panjang lengkung Henle
yang berbeda. Berdasarkan penemuan tersebut, hipotesis yang mungkin dibangun adalah:
A. ketiga rodensia memiliki laju filtrasi ginjal yang berbeda-beda
B. ketiga rodensia memiliki kepekatan urin yang berbeda-beda
C. ketiga rodensia menghasilkan urin dengan pH yang berbeda-beda
D. ketiga rodensia memproduksi urin dengan volume yang sama
E. ketiga rodensia hidup pada habitat yang sama

11
03-OSK-2011
Copyright © 2011 by Kemdiknas
Hak Cipta dilindungi oleh Undang-undang
Nama: .........................................................................................................
Asal SMA/Kelas: ........................................................................................

Seleksi Calon Peserta IBO 2012
Tingkat Kabupaten/Kota (Kode:03)

16. Sel-sel dari kelenjar berikut mensekresikan hormon yang berperan meningkatkan kadar gula
darah dan mempercepat kerja jantung serta pernafasan ketika tubuh sedang dalam kondisi
tercekam. Kelenjar yang dimaksud adalah:
A. pituitari
B. adrenal
C. pankreas
D. tiroid
E. hipothalamus
17. Perhatikan gambar berikut ini:

Seorang dokter dapat memeriksa kondisi kehidupan dari pasiennya dengan memberikan
sorotan lampu senter ke arah mata. Pada orang yang masih hidup, pemberian cahaya akan
direspon melalui perubahan diameter bagian (......). Akan tetapi, pada orang mati bagian
tersebut tidak akan merespon terhadap cahaya senter.
A. A
B. B
C. C
D. D
E. E
18. Berdasarkan bentuknya, karakter yang tepat untuk mewakili sel saraf berikut adalah:

12
03-OSK-2011
Copyright © 2011 by Kemdiknas
Hak Cipta dilindungi oleh Undang-undang
Nama: .........................................................................................................
Asal SMA/Kelas: ........................................................................................

A.
B.
C.
D.
E.

Seleksi Calon Peserta IBO 2012
Tingkat Kabupaten/Kota (Kode:03)

berperan dalam fungsi sensoris dan ditemukan di sistem saraf pusat
berperan dalam fungsi motoris dan ditemukan di sistem saraf tepi
berperan dalam fungsi sensoris dan ditemukan dalam sistem saraf tepi
berperan dalam fungsi motoris dan ditemukan dalam sistem saraf pusat
berperan dalam fungsi integrasi dan ditemukan di sistem saraf pusat

19. Sistem saraf pada manusia di bagi menjadi sistem saraf pusat dan sistem saraf tepi (perifer).
Manakah dari pernyataan berikut ini yang tepat mengenai sistem saraf tepi?
A. Sistem saraf tepi terletak di otak dan susum tulang belakang
B. Sistem saraf tepi mengandung sel-sel saraf yang merupakan perpanjangan dari sistem
saraf pusat
C. Sistem saraf tepi mengandung interneuron dari sistem saraf pusat
D. Sistem saraf tepi merupakan sistem saraf pusat yang menjadi bagian dari susum lanjutan
E. Sistem saraf tepi merupakan pusat pengaturan sistem reflek
20. Salah satu indikasi terjadinya infeksi di dalam tubuh adalah demam. Demam timbul karena
produksi pirogen oleh sel darah putih pada saat infeksi terjadi. Pirogen mempengaruhi ..... di
otak untuk meningkatkan nilai acuan suhu tubuh dari 37 ke suhu yang lebih tinggi. Usaha
tubuh untuk meningkatkan suhu tubuh inilah yang menimbulkan sensasi kedinginan pada saat
demam. Pilihlah jawaban yang tepat untuk melengkapi keterangan di atas.
A. Hipofisa
B. Hipotalamus
C. Epifisa
D. Sistem limbik
E. Korteks otak besar
21. Ketika seseorang yang alergi seruk sari terpapar dengan serbuk sari, matanya akan berair dan
timbul bintik-bintik merah di permukaan kulitnya. Peristiwa ini terjadi akibat pelepasan:
A. antigen oleh sel darah merah
B. enzim oleh platelet
C. histamin oleh sel-sel mast
D. hormon oleh kelenjar pituitari
E. mukus oleh sel-sel eksokrin
22. Perhatikan diagram perkembangan berikut ini.

Istilah yang tepat untuk menggambarkan A dan B adalah:
A. A: Ovulasi, B : Diferensiasi
B. A: Meiosis, B : Organogenesis
13
03-OSK-2011
Copyright © 2011 by Kemdiknas
Hak Cipta dilindungi oleh Undang-undang
Nama: .........................................................................................................
Asal SMA/Kelas: ........................................................................................

Seleksi Calon Peserta IBO 2012
Tingkat Kabupaten/Kota (Kode:03)

C. A: Proses pembelahan (cleavage), B: Diferensiasi
D. A: Germinasi, B: Diferensiasi
E. A: Morulasi, B: Gastrulasi
23. Perbedaan mendasar antara tulang dan rawan adalah bahwa rawan:
A. merupakan salah satu tipe jaringan ikat
B. tidak memiliki pembuluh darah dan saraf
C. mensekresi matriks padat
D. tersusun atas kolagen dan garam
E. merupakan bagian dari sistem rangka
24. Komponen berikut ini yang konsentrasinya meningkat di sitoplasma pada saat otot
berkontraksi adalah:
A. ATP
B. Ca2+
C. Mg2+
D. Aktin
E. Myosin
25. Berikut ini adalah urutan kejadian yang terjadi pada otot lurik.

Molekul apakah yang diwakili oleh X?
A. hemoglobin
B. glikogen
C. asam amino
D. alkohol
E. asam laktat

14
03-OSK-2011
Copyright © 2011 by Kemdiknas
Hak Cipta dilindungi oleh Undang-undang
Nama: .........................................................................................................
Asal SMA/Kelas: ........................................................................................

Seleksi Calon Peserta IBO 2012
Tingkat Kabupaten/Kota (Kode:03)

ETOLOGI (Nilai 5)
1.

Penjelasan yang paling baik untuk menjelaskan perilaku manusia yang tersenyum pada saat
menyapa teman walaupun mereka berasal dari komunitas yang berbeda adalah:
A. Mereka memiliki beberapa kultur yang umum
B. Mereka mengingat wajah yang tersenyum pada saat mereka masih bayi
C. Mereka telah mempelajari bahwa senyum tidak memicu perilaku agresif
D. Senyum merupakan suatu pola perilaku yang diturunkan secara genetis
E. Senyum adalah suatu perilaku yang dipelajari

2.

Merkat merupakan hewan sosial yang hidup di daearah savana di Afrika. Hewan ini hidup
pada sistem terowongan di dalam tanah yang memungkinkan mereka untuk melarikan diri
dari pemangsa. Untuk mengetahui kedatangan pemangsa, koloni Merkat mengandalkan
kepada sekelompok kecil individu yang menjadi pengintai di luar sistem koloni. Pada saat
pemangsa datang, pengintai ini mengeluarkan suara peringatan sampai seluruh anggota
koloni aman di dalam sistem terowongan. Perilaku ini merupakan salah satu contoh dari:
A. social learning
B. conditional learning
C. association learning
D. alturism
E. imprinting

3.

Karl von Firsch merupakan salah satu orang penting dalam dunia penelitian perilaku hewan.
Beliau berhasil menerjemahkan bahasa lebah pekerja yang memberikan informasi posisi
sumber makanan. Bahasa lebah tersebut dikenal dengan istilah round dance dan waggle
dance. Hasil penelitian Karl von Firsch tersebut menunjukkan peranan (.....) dalam
menentukan perilaku hewan.
A. belajar
B. tanda dan stimulus
C. hadiah dan hukuman
D. pengulangan stimulus
E. hubungan sosial

4.

Pada hewan, perilaku ritual dengan sedikit risiko kecelakaan serius atau kematian pada
pelakunya pada populasi adalah:
A. hirarki dominansi yang stabil
B. altruisme biologis
C. radiasi adaptif
D. perilaku insting
E. habituasi

5.

Burung hitam Eropa terkenal dengan kemampuannya untuk melakukan migrasi yang
terkondisi. Hal ini dipicu ketika:
A. persaingan antara setiap individu mulai semakin parah.
B. kondisi musim memungkinkan mereka untuk melakukan navigasi berdasarkan kepada
pola-pola yang terdapat di langit.
C. keuntungan dari mengkolonisasi habitat baru melebihi bahaya yang didapatkan dalam
perjalanan menuju ke lokasi tersebut
D. jumlah dari individu-individu yang dominan secara sosial menurun tajam.
E. semua pernyataan di atas salah
15
03-OSK-2011
Copyright © 2011 by Kemdiknas
Hak Cipta dilindungi oleh Undang-undang
Nama: .........................................................................................................
Asal SMA/Kelas: ........................................................................................

Seleksi Calon Peserta IBO 2012
Tingkat Kabupaten/Kota (Kode:03)

GENETIKA DAN EVOLUSI (Nilai 20)
1.

Hukum Mendel pertama menyatakan bahwa unit-unit penurunan sifat yang mengontrol
fenotip berbeda dapat diturunkan secara terpisah. Manakah dari pernyataan berikut yang
paling sesuai dengan bunyi hukum tersebut?
A. Mutasi merupakan kejadian acak yang mempengaruhi setiap lokus secara terpisah
B. Saat fertilisasi, sperma mana yang akan membuahi sel telur merupakan peristiwa acak
C. Saat meiosis I, kromosom berpasangan dengan homolog
D. Saat meiosis I, pemisahan satu pasangan kromosom tidak mempengaruhi pemisahan
pasangan kromosom lainnya
E. Saat mitosis, tidak terdapat perbedaan antara DNA induk dengan DNA hasil replikasi

2.

Pada jenis ayam tertentu, gen untuk bulu hitam dan bulu putih bersifat kodominan. Ayam
jenis ini akam memiliki:
A. tiga tipe fenotip untuk warna bulu
B. dua fenotip untuk warna bulu
C. hanya dua genotip untuk warna bulu
D. hanya memiliki bulu bewarna hitam atau putih saja
E. tiga genotip untuk bulu warna hitam

3.

Seorang peneliti melakukan persilangan diantara dua tikus yang keduanya memiliki rambut
hitam. Rambut hitam dominan terhadap putih. Sebanyak 75% keturunan berambut hitam dan
sisanya berambut putih. Peneliti dapat berasumsi bahwa genotip parental paling mungkin
adalah:
A. BB x BB
B. BB x Bb
C. BB x bb
D. Bb x Bb
E. bb x bb

4.

Suatu sel abnormal memiliki pasangan kromosom yang dinyatakan dengan A1/A2 B1/B1 C1/C2
D1, dimana kromosom D tidak memiliki pasangan homolog. Berdasarkan tipe kromosom
penyusunnya, berapa jenis sel gamet yang anda harap dapat dihasilkan dari turunan sel
tersebut?
A. 8
B. 7
C. 6
D. 4
E. 2

5.

Saat ini banyak sekali orang takut terkena penyakit kanker jika memiliki orang tua yang
terserang kanker. Salah satu penyebab timbulnya kanker adalah akibat adanya mutasi pada
gen-gen yang mengatur siklus sel. Ketakutan dari orang-orang tersebut beralasan apabila gengen yang termutasi ditemukan pada (.....) dari orang tuanya.
A. sel hati
B. sel pankreas
C. sel saraf
D. sel-sel yang berada di dalam tubulus seminiferus
E. sel-sel penyusun jaringan ikat di ovarium
16
03-OSK-2011
Copyright © 2011 by Kemdiknas
Hak Cipta dilindungi oleh Undang-undang
Nama: .........................................................................................................
Asal SMA/Kelas: ........................................................................................

6.

Seleksi Calon Peserta IBO 2012
Tingkat Kabupaten/Kota (Kode:03)

Sifat tinggi batang dari tanaman X diatur secara poligenik oleh dua alel (A dan B) yang
terdapat pada kromosom berbeda. Tanaman dengan genotip AABB memiliki tinggi 18 cm,
sedangkan tanaman dengan genotipe aabb memiliki tinggi 10 cm. Jika persilangan antara dua
tanaman AaBb x AaBb menghasilkan 160 keturunan. Berapa banyak dari keturunan tersebut
diharapkan memiliki tinggi 16 cm?
A. 150
B. 90
C. 60
D. 30
E. 10
Untuk pertanyaan No.7-8: Perhatikan pohon silsilah berikut ini. Warna abu-abu
menunjukkan individu yang menderita suatu penyakit.
I

1

2

2

3

II
1

4

5

III
1

7.

2

3

4

IV
V
1
2
V
Berdasarkan pohon silslah di atas, bagaimanakah pola penurunan penyakit ini?
A.
B.
C.
D.
E.

Autosomal resesif
Autosomal dominan
Terpaut X dominan
Terpaut X resesif
Terpaut Y

8.

Jika IV-1 kawin dengan seorang laki-laki yang tidak menderita penyakit tersebut, berapa
peluang ia akan melahirkan anak laki-laki yang sehat?
A. 0 %
B. 25 %
C. 50 %
D. 75 %
E. 100 %

9.

Pada Drosophila melanogaster, alel dominan (w+) mengatur munculnya sifat mata merah,
sedangkan alel resesif (w) yang merupakan alel mutan, mengatur munculnya sifat mata putih.
Alel ini terpaut kromosom X. Jika perkawinan antar dua Drosophila menghasilkan keturunan
½ mata putih dan ½ mata merah, maka genotip dari kedua Drosophila tersebut adalah:
A. Xw+Xw+ vs. XwY
B. XwXw+ vs. Xw+Y
C. Xw+Xw vs. XwY
D. XwXw vs. Xw+Y
E. XwXw vs. XwY
17
03-OSK-2011
Copyright © 2011 by Kemdiknas
Hak Cipta dilindungi oleh Undang-undang
Nama: .........................................................................................................
Asal SMA/Kelas: ........................................................................................

Seleksi Calon Peserta IBO 2012
Tingkat Kabupaten/Kota (Kode:03)

10. Sebuah mutasi nonsense terjadi di tengah urutan coding gen M. Apakah yang anda harapkan
terjadi pada ukuran produk transkripsi (panjang basa) dan translasi (Dalton) dari gen M
tersebut dibanding produk akhir dari gen M normal?
A. Produk transkripsi memiliki ukuran yang lebih kecil, sedangkan produk translasi tidak
berubah
B. Produk transkripsi tidak berubah, sedangkan produk translasi memiliki ukuran yang lebih
kecil
C. Produk transkripsi dan translasi memiliki ukuran yang lebih kecil
D. Produk transkripsi dan translasi tidak berubah
E. Tidak dihasilkan produk transkripsi dan translasi
11. Suatu populasi yang berada dalam kesetimbangan Hardy-Weinberg memiliki dua alel untuk
suatu lokus, A dan a. Jika frekuensi A sebesar 0,7; berapa frekuensi individu heterozigot pada
populasi tersebut?
A. 0,09
B. 0,42
C. 0,49
D. 0,51
E. 0,91
12. Suatu populasi berada dalam kesetimbangan genetik ketika frekuensi alel-alel dari suatu gen
pada populasi tersebut tetap sama dari satu generasi ke generasi berikutnya. Kesetimbangan
genetik akan lebih mudah tercapai apabila:
A. populasi berukuran kecil sehingga lebih rentan terhadap genetic drift
B. populasi memperoleh mutasi yang menguntugkan
C. populasi tidak mengalami imigrasi dan emigrasi
D. terjadi perkawinan antar kerabat dekat
E. seleksi alam bekerja terhadap fenotip tertentu
Untuk pertanyaan No.13-14: Perhatikan gambar berikut ini.

13. Manakah dari kurva di atas yang paling baik menggambarkan konsep seleksi penstabilan
(stabilizing selection) atau seleksi yang lebih menyukai varian intermediet daripada individuindividu dengan fenotip ekstrim?
A. Hanya grafik I
B. Hanya grafik II
C. Grafik I dan grafik II
D. Hanya grafik III
E. Grafik II dan grafik III
18
03-OSK-2011
Copyright © 2011 by Kemdiknas
Hak Cipta dilindungi oleh Undang-undang
Nama: .........................................................................................................
Asal SMA/Kelas: ........................................................................................

Seleksi Calon Peserta IBO 2012
Tingkat Kabupaten/Kota (Kode:03)

14. Manakah dari kurva di atas yang paling baik menggambarkan konsep seleksi diversifikasi
atau tipe seleksi yang lebih menyukai fenotip ekstrim daripada fenotip intermediet?
A. Hanya grafik I
B. Hanya grafik II
C. Grafik I dan grafik II
D. Hanya grafik III
E. Grafik II dan grafik III
15. Sebelum kedatangan bangsa Eropa, Amerika Utara merupakan habitat bagi jutaan ayam
padang rumput (Tympanuchus cupido). Akibat perburuan liar dan perusakan habitat, ukuran
populasi mereka menurun drastis hingga hampir mencapai kepunahan. Keanekaragaman
genetik ayam padang rumput ikut berkurang sehingga kesuksesan reproduktif mereka
menurun. Mekanisme evolusi apakah yang terjadi pada populasi tersebut?
A. Aliran gen (gene flow)
B. Genetic drift
C. Mutasi
D. Perkawinan tidak acak
E. Seleksi buatan
16. Walaupun anjing laut dan penguin sama-sama memiliki tubuh ramping yang cocok untuk
berenang dan dilapisi oleh lemak, kedua hewan ini tidak berkerabat dekat. Kesamaan ini
dihasilkan oleh:
A. koevolusi
B. evolusi konvergen
C. radiasi adaptif
D. seleksi terarah
E. spesiasi
17. Pada kondisi tertentu, suatu populasi dapat berkembang menjadi dua atau lebih spesies tanpa
keberadaan pemisah geografis antara lain dengan perbedaan pada perilaku. Perilaku yang
terdapat di bawah ini dapat memicu terjadinya spesiasi, KECUALI:
A. seleksi dari buah yang menjadi inang secara spesifik oleh lalat buah
B. perilaku kawin berbeda yang dikembangkan oleh burung layang-layang pada habitat
yang berbeda
C. pemilihan waktu untuk berkembang biak oleh kupu-kupu
D. pelepasan gamet ke lautan oleh invertebrate laut sebagai bentuk respons terhadap
senyawa kimia pada air
E. pemilihan tipe mangsa oleh elang yang memiliki kisaran mangsa yang luas
18. Dua populasi bekicot terisolasi secara baik dalam jangka waktu panjang. Berdasarkan kepada
konsep spesies, manakah dari pernyataan berikut yang menunjukkan bahwa kedua populasi
ini telah menjadi dua spesies yang berbeda?
A. Kedua populasi memiliki sekurang-kurangnya lima sifat morfologis yang berbeda
B. Kedua populasi memiliki urutan DNA yang berbeda
C. Kedua populasi memiliki perbedaan kandungan protein
D. Kedua populasi memiliki respon yang berbeda pada saat terkena pestisida dengan dosis
yang sama
E. Hibrid steril akan dihasilkan ketika anggota dari kedua populasi tersebut disilangkan

19
03-OSK-2011
Copyright © 2011 by Kemdiknas
Hak Cipta dilindungi oleh Undang-undang
Nama: .........................................................................................................
Asal SMA/Kelas: ........................................................................................

Seleksi Calon Peserta IBO 2012
Tingkat Kabupaten/Kota (Kode:03)

19. Bipedalisme diyakini telah berevolusi pada garis nenek moyang manusia karena pergerakan
bipedal:
A. lebih efisien daripada pergerakan quadrupedal, karena dapat memberi kebebasan bagi
anggota gerak depan untuk memanipulasi objek
B. lebih efisien daripada pergerakan quadrupedal karena energi yang diperlukan lebih
sedikit
C. kurang efisien daripada gerakan quadrupedal, tetapi dapat memberi kebebasan bagi
anggota gerak depan untuk memanipulasi objek
D. kurang efisien daripada gerakan quadrupedal, tetapi hewan bipedal dapat berlari lebih
kencang
E. kurang efisien daripada gerakan quadrupedal, tetapi seleksi alam tidak bekerja untuk
memperbaiki efisiensi
20. Pada awal kehidupan, organisme hanya ditemukan di lautan. Namun dengan berkembangnya
kerak benua, dan terdapatnya daratan yang luas, makhluk hidup mulai menginvasi daratan.
Begitu pula dengan tumbuhan. Pada tumbuhan, organ yang terbentuk sebelum menginvasi
daratan adalah:
A. dinding sel
B. kutikula
C. spora
D. akar
E. stomata

20
03-OSK-2011
Copyright © 2011 by Kemdiknas
Hak Cipta dilindungi oleh Undang-undang
Nama: .........................................................................................................
Asal SMA/Kelas: ........................................................................................

Seleksi Calon Peserta IBO 2012
Tingkat Kabupaten/Kota (Kode:03)

EKOLOGI (Nilai 10)
Untuk pertanyaan No.1-2: Perhatikan grafik pertumbuhan penduduk dari lima negara
berikut ini.

1.

Negara mana yang mengalami petumbuhan penduduk paling pesat?
A. (A)
B. (B)
C. (C)
D. (D)
E. (E)

2.

Negara mana yang memiliki petumbuhan penduduk mendekati nol?
A. (A)
B. (B)
C. (C)
D. (D)
E. (E)

3.

Mimikri adalah salah satu strategi yang dihasilkan sebagai proses dari seleksi alami yang
berperan besar dalam meningkatkan kelulushidupan dari suatu organisme. Manakah dari
pernyataan berikut ini yang tergolong sebagai bentuk dari mimikri Batesian?
A. Lipan yang memiliki senyawa racun bagi katak dan selalu dihindari oleh katak setelah
katak terkena racun lipan tersebut pada saat proses pemangsaan sebelumnya
B. Ngengat yang memiliki gambar seperti “mata palsu” pada bagian sayap belakang yang
berperan untuk membingungkan musuhnya
C. Anak camar yang memiliki warna serupa dengan lingkungan tempat dia hidup
D. Anak camar yang dapat memperoleh makanan dari burung selain induknya dengan
mengetuk titik yang tepat pada paruh dari burung tersebut
E. Ngengat yang memiliki warna serupa dengan lebah penyengat

4.

Energi yang tersimpan dalam organisme pada suatu tingkatan trofik tertentu adalah:
A. kira-kira 10% dari energi pada tingkatan trofik di bawahnya
B. kira-kira 10% dari energi pada tingkatan trofik di atasnya
C. kira-kira 50% dari energi pada tingkatan trofik di bawahnya
D. kira-kira 50% dari energi pada tingkatan trofik di atasnya
E. 100% dari energi pada tingkatan trofik di bawahnya

21
03-OSK-2011
Copyright © 2011 by Kemdiknas
Hak Cipta dilindungi oleh Undang-undang
Nama: .........................................................................................................
Asal SMA/Kelas: ........................................................................................

Seleksi Calon Peserta IBO 2012
Tingkat Kabupaten/Kota (Kode:03)

5.

Pada tingkatan konsumen primer, proses utama yang mengurangi energi sehingga tidak
menjadi biomassa yang dapat dimanfaatkan pada tingkatan di atasnya adalah:
A. fotosintesis
B. ekskresi
C. pertumbuhan
D. perkembangan
E. respirasi

6.

Diantara kelompok organisme berikut, manakah yang dapat berperan sebagai konsumen
sekunder maupun tersier?
A. Dekomposer
B. Herbivora
C. Detritivora
D. Autotrof
E. Karnivora

7.

Kelompok tumbuhan klimaks pada proses suksesi sekunder terestrial umumnya berasal dari
kelompok:
a. rumput
b. herba
c. perdu dan semak
d. pohon berkayu keras
e. pohon konifer

8.

Manakah dari pernyataan berikut ini yang tepat mengenai siklus hidrologi?
A. Karena air diputarkan dalam siklus, maka manusia tidak akan pernah kehabisan air tawar
untuk dimanfaatkan.
B. Sebagian air mengalami evaporasi dari daratan dan dari tumbuhan
C. Semua molekul air yang dievaporasi dari lautan akan jatuh sebagai hujan di daratan
D. Seluruh air yang turun sebagai hujan akan mengalami gaya gravitasi sehingga akan
terbawa kembali ke laut
E. Setelah terserap ke dalam tanah, air menjadi aman dari eksploitasi dan pencemaran oleh
manusia

9.

Tumbuhan mengambil nitrogen dari dalam tanah dalam bentuk molekul:
A. N2
B. NO
C. NO2
D. NO3E. NO4+

10. Energi dalam suatu ekosistem mengalir tetapi tidak diputarkan (tidak membentuk siklus)
karena:
A. energi hilang (musnah) setelah terpakai
B. energi tersebar merata pada semua organisme
C. energi dikonversi menjadi berbagai bentuk energi yang bermanfaat
D. energi meningkat sejalan dengan naiknya tingkatan trofik
E. energi tidak dapat digunakan jika sudah terkonversi menjadi panas

22
03-OSK-2011
Copyright © 2011 by Kemdiknas
Hak Cipta dilindungi oleh Undang-undang
Nama: .........................................................................................................
Asal SMA/Kelas: ........................................................................................

Seleksi Calon Peserta IBO 2012
Tingkat Kabupaten/Kota (Kode:03)

BIOSISTEMATIK (Nilai 5)
1.

Berikut ini manakah yang merujuk pada taksa tingkat famili dari organisme?
A. Solanum
B. Pandanales
C. Felidae
D. Ipomoea
E. Glycine

2.

Kelompok tumbuhan yang memiliki biji yang tertutup oleh daun buah (perikarp) adalah:
A. Monokotil
B. Dikotil
C. Angiospermae
D. Gymnospermae
E. Spermatophyta

3.

Salah satu argumentasi yang menyebabkan jamur (fungi) di kelompokkan ke dalam satu
kerajaan khusus adalah:
I. Jamur tidak memiliki klorofil
II. Jamur dapat berkembang biak secara generatif dan vegetatif
III. Jamur merupakan organisme heterotrof
IV. Jamur berkembang biak dengan spora
V. Bagian-bagian jamur tidak dapat dibedakan antara daun, batang dan akar
Manakah dari kombinasi pernyataan di atas yang benar?
A. I, II, III
B. II, III, IV
C. III, IV, V
D. I, III
E. I, V

4.

Dalam daur hidup katak, stadium yang paling muda ditandai dengan:
A. terbentuknya kaki depan
B. terbentuknya kaki belakang
C. menghilangnya ekor
D. bernafas dengan insang luar
E. bernafas dengan insang dalam

5.

Organisme berikut ini termasuk kelas Mammalia yang hidup di perairan, KECUALI:
A. Pesut
B. Paus
C. Kuda Laut
D. Dugong
E. Lumba-lumba

23
03-OSK-2011
Copyright © 2011 by Kemdiknas
Hak Cipta dilindungi oleh Undang-undang

More Related Content

What's hot

laporan praktikum hidrokarbon
laporan praktikum hidrokarbonlaporan praktikum hidrokarbon
laporan praktikum hidrokarbonwd_amaliah
 
kumpulan soal hukum-hukum gas
kumpulan soal hukum-hukum gaskumpulan soal hukum-hukum gas
kumpulan soal hukum-hukum gasRfebiola
 
spektrofotometri uv-vis
spektrofotometri uv-visspektrofotometri uv-vis
spektrofotometri uv-visHafifa Marza
 
FISIKA - AYUNAN SEDERHANA
FISIKA - AYUNAN SEDERHANAFISIKA - AYUNAN SEDERHANA
FISIKA - AYUNAN SEDERHANAPRAMITHA GALUH
 
Laporan Biologi Fermentasi
Laporan Biologi Fermentasi Laporan Biologi Fermentasi
Laporan Biologi Fermentasi Hilya Auliya
 
Soal Laju Reaksi + Pembahasan
Soal Laju Reaksi + PembahasanSoal Laju Reaksi + Pembahasan
Soal Laju Reaksi + PembahasanArsyi Nurani
 
Perkawinan trihibrid
Perkawinan trihibridPerkawinan trihibrid
Perkawinan trihibridJeneng Omega
 
Laporan Pengaruh Cahaya Terhadap Pertumbuhan Kacang Hijau
Laporan Pengaruh Cahaya Terhadap Pertumbuhan Kacang HijauLaporan Pengaruh Cahaya Terhadap Pertumbuhan Kacang Hijau
Laporan Pengaruh Cahaya Terhadap Pertumbuhan Kacang Hijauanurputri
 
Pengaruh media tanam pada perkembangan kacang hijau
Pengaruh media tanam pada perkembangan kacang hijauPengaruh media tanam pada perkembangan kacang hijau
Pengaruh media tanam pada perkembangan kacang hijauAyik Novitasari
 
Laporan percobaan enzim katalase
Laporan percobaan enzim katalase Laporan percobaan enzim katalase
Laporan percobaan enzim katalase DaPiDaBi
 
LAPORAN PRAKTIKUM PENGENALAN ALAT-ALAT MIKROBIOLOGI
LAPORAN PRAKTIKUM PENGENALAN ALAT-ALAT MIKROBIOLOGILAPORAN PRAKTIKUM PENGENALAN ALAT-ALAT MIKROBIOLOGI
LAPORAN PRAKTIKUM PENGENALAN ALAT-ALAT MIKROBIOLOGIEDIS BLOG
 
1 b 11170163000059_laporan_momentum dan impuls
1 b 11170163000059_laporan_momentum dan impuls1 b 11170163000059_laporan_momentum dan impuls
1 b 11170163000059_laporan_momentum dan impulsumammuhammad27
 

What's hot (20)

Massa jenis zat cair
Massa jenis zat cairMassa jenis zat cair
Massa jenis zat cair
 
laporan praktikum hidrokarbon
laporan praktikum hidrokarbonlaporan praktikum hidrokarbon
laporan praktikum hidrokarbon
 
kumpulan soal hukum-hukum gas
kumpulan soal hukum-hukum gaskumpulan soal hukum-hukum gas
kumpulan soal hukum-hukum gas
 
Uji Kelarutan Lemak
Uji Kelarutan LemakUji Kelarutan Lemak
Uji Kelarutan Lemak
 
Laporan fermentasi pembuatan yoghurt
Laporan fermentasi pembuatan yoghurtLaporan fermentasi pembuatan yoghurt
Laporan fermentasi pembuatan yoghurt
 
spektrofotometri uv-vis
spektrofotometri uv-visspektrofotometri uv-vis
spektrofotometri uv-vis
 
FISIKA - AYUNAN SEDERHANA
FISIKA - AYUNAN SEDERHANAFISIKA - AYUNAN SEDERHANA
FISIKA - AYUNAN SEDERHANA
 
Laporan Biologi Fermentasi
Laporan Biologi Fermentasi Laporan Biologi Fermentasi
Laporan Biologi Fermentasi
 
Soal Laju Reaksi + Pembahasan
Soal Laju Reaksi + PembahasanSoal Laju Reaksi + Pembahasan
Soal Laju Reaksi + Pembahasan
 
MUTASI pada GENOM
MUTASI pada GENOMMUTASI pada GENOM
MUTASI pada GENOM
 
Entropi (new)
Entropi (new)Entropi (new)
Entropi (new)
 
Perkawinan trihibrid
Perkawinan trihibridPerkawinan trihibrid
Perkawinan trihibrid
 
Laporan Pengaruh Cahaya Terhadap Pertumbuhan Kacang Hijau
Laporan Pengaruh Cahaya Terhadap Pertumbuhan Kacang HijauLaporan Pengaruh Cahaya Terhadap Pertumbuhan Kacang Hijau
Laporan Pengaruh Cahaya Terhadap Pertumbuhan Kacang Hijau
 
Pengaruh media tanam pada perkembangan kacang hijau
Pengaruh media tanam pada perkembangan kacang hijauPengaruh media tanam pada perkembangan kacang hijau
Pengaruh media tanam pada perkembangan kacang hijau
 
Laporan percobaan enzim katalase
Laporan percobaan enzim katalase Laporan percobaan enzim katalase
Laporan percobaan enzim katalase
 
LAPORAN PRAKTIKUM PENGENALAN ALAT-ALAT MIKROBIOLOGI
LAPORAN PRAKTIKUM PENGENALAN ALAT-ALAT MIKROBIOLOGILAPORAN PRAKTIKUM PENGENALAN ALAT-ALAT MIKROBIOLOGI
LAPORAN PRAKTIKUM PENGENALAN ALAT-ALAT MIKROBIOLOGI
 
Laporan enzim katalase
Laporan enzim katalaseLaporan enzim katalase
Laporan enzim katalase
 
Laporan Praktikum Laju Reaksi
Laporan Praktikum Laju ReaksiLaporan Praktikum Laju Reaksi
Laporan Praktikum Laju Reaksi
 
Pengaruh cahaya bagi pertumbuhan tanaman kacang hijau
Pengaruh cahaya bagi pertumbuhan tanaman kacang hijauPengaruh cahaya bagi pertumbuhan tanaman kacang hijau
Pengaruh cahaya bagi pertumbuhan tanaman kacang hijau
 
1 b 11170163000059_laporan_momentum dan impuls
1 b 11170163000059_laporan_momentum dan impuls1 b 11170163000059_laporan_momentum dan impuls
1 b 11170163000059_laporan_momentum dan impuls
 

Viewers also liked

Soal osn biologi smp 2012 kabupaten
Soal osn biologi smp 2012 kabupatenSoal osn biologi smp 2012 kabupaten
Soal osn biologi smp 2012 kabupatenWayan Sudiarta
 
Soal osn-biologi-tingkat-kabupaten-kota-tahun-2015
Soal osn-biologi-tingkat-kabupaten-kota-tahun-2015Soal osn-biologi-tingkat-kabupaten-kota-tahun-2015
Soal osn-biologi-tingkat-kabupaten-kota-tahun-2015rina72
 
Contoh soal latihan biologi dan jawabannya
Contoh soal latihan biologi dan jawabannyaContoh soal latihan biologi dan jawabannya
Contoh soal latihan biologi dan jawabannyaRisca Wentiari
 
Soal OSN IPA SMP 2016.pdf
Soal OSN  IPA SMP 2016.pdfSoal OSN  IPA SMP 2016.pdf
Soal OSN IPA SMP 2016.pdfWahyudi Oetomo
 
Soal osn-biologi-tingkat-kabupaten-kota-tahun-2014
Soal osn-biologi-tingkat-kabupaten-kota-tahun-2014Soal osn-biologi-tingkat-kabupaten-kota-tahun-2014
Soal osn-biologi-tingkat-kabupaten-kota-tahun-2014Entertainment
 
Penyelesaian soal osn fisika smp 2010 tingkat kabupaten
Penyelesaian soal osn fisika smp 2010 tingkat kabupatenPenyelesaian soal osn fisika smp 2010 tingkat kabupaten
Penyelesaian soal osn fisika smp 2010 tingkat kabupatenWayan Sudiarta
 
KUMPULAN SOAL PREDIKSI OLIMPIADE BIOLOGI UDAYANA
KUMPULAN SOAL PREDIKSI OLIMPIADE BIOLOGI UDAYANA KUMPULAN SOAL PREDIKSI OLIMPIADE BIOLOGI UDAYANA
KUMPULAN SOAL PREDIKSI OLIMPIADE BIOLOGI UDAYANA Nita Sulistyawati
 
Latihan soal olimpiade fisika SMP
Latihan soal olimpiade fisika SMPLatihan soal olimpiade fisika SMP
Latihan soal olimpiade fisika SMPDaniel Tohari
 
Prediksi unas ipa smp 2013 paket 1
Prediksi unas ipa smp 2013 paket 1Prediksi unas ipa smp 2013 paket 1
Prediksi unas ipa smp 2013 paket 1Koran rakyat
 
PENERAPAN TEKNOLOGI BUDIDAYA JENUH AIR PADA TANAMAN PADI DAN KEDELAI UNTUK ME...
PENERAPAN TEKNOLOGI BUDIDAYA JENUH AIR PADA TANAMAN PADI DAN KEDELAI UNTUK ME...PENERAPAN TEKNOLOGI BUDIDAYA JENUH AIR PADA TANAMAN PADI DAN KEDELAI UNTUK ME...
PENERAPAN TEKNOLOGI BUDIDAYA JENUH AIR PADA TANAMAN PADI DAN KEDELAI UNTUK ME...Repository Ipb
 
Evaluasi SBO
Evaluasi SBOEvaluasi SBO
Evaluasi SBOmutisav
 

Viewers also liked (20)

SOAL OSK BIOLOGI 2013
SOAL OSK BIOLOGI 2013SOAL OSK BIOLOGI 2013
SOAL OSK BIOLOGI 2013
 
Soal osn biologi smp 2012 kabupaten
Soal osn biologi smp 2012 kabupatenSoal osn biologi smp 2012 kabupaten
Soal osn biologi smp 2012 kabupaten
 
Soal OSK Biologi SMA 2016
Soal OSK Biologi SMA 2016Soal OSK Biologi SMA 2016
Soal OSK Biologi SMA 2016
 
Soal osn-biologi-tingkat-kabupaten-kota-tahun-2015
Soal osn-biologi-tingkat-kabupaten-kota-tahun-2015Soal osn-biologi-tingkat-kabupaten-kota-tahun-2015
Soal osn-biologi-tingkat-kabupaten-kota-tahun-2015
 
Contoh soal latihan biologi dan jawabannya
Contoh soal latihan biologi dan jawabannyaContoh soal latihan biologi dan jawabannya
Contoh soal latihan biologi dan jawabannya
 
Soal osn ipa 2015
Soal osn ipa 2015Soal osn ipa 2015
Soal osn ipa 2015
 
Biologi forensik
Biologi forensikBiologi forensik
Biologi forensik
 
Soal OSN IPA SMP 2016.pdf
Soal OSN  IPA SMP 2016.pdfSoal OSN  IPA SMP 2016.pdf
Soal OSN IPA SMP 2016.pdf
 
Soal smp
Soal smpSoal smp
Soal smp
 
Soal osn-biologi-tingkat-kabupaten-kota-tahun-2014
Soal osn-biologi-tingkat-kabupaten-kota-tahun-2014Soal osn-biologi-tingkat-kabupaten-kota-tahun-2014
Soal osn-biologi-tingkat-kabupaten-kota-tahun-2014
 
Penyelesaian soal osn fisika smp 2010 tingkat kabupaten
Penyelesaian soal osn fisika smp 2010 tingkat kabupatenPenyelesaian soal osn fisika smp 2010 tingkat kabupaten
Penyelesaian soal osn fisika smp 2010 tingkat kabupaten
 
Soal smp
Soal smpSoal smp
Soal smp
 
Silabus OSN SMP 2016
Silabus OSN SMP 2016Silabus OSN SMP 2016
Silabus OSN SMP 2016
 
KUMPULAN SOAL PREDIKSI OLIMPIADE BIOLOGI UDAYANA
KUMPULAN SOAL PREDIKSI OLIMPIADE BIOLOGI UDAYANA KUMPULAN SOAL PREDIKSI OLIMPIADE BIOLOGI UDAYANA
KUMPULAN SOAL PREDIKSI OLIMPIADE BIOLOGI UDAYANA
 
Juklak osn smp_2016
Juklak osn smp_2016Juklak osn smp_2016
Juklak osn smp_2016
 
Latihan soal olimpiade fisika SMP
Latihan soal olimpiade fisika SMPLatihan soal olimpiade fisika SMP
Latihan soal olimpiade fisika SMP
 
Prediksi unas ipa smp 2013 paket 1
Prediksi unas ipa smp 2013 paket 1Prediksi unas ipa smp 2013 paket 1
Prediksi unas ipa smp 2013 paket 1
 
PENERAPAN TEKNOLOGI BUDIDAYA JENUH AIR PADA TANAMAN PADI DAN KEDELAI UNTUK ME...
PENERAPAN TEKNOLOGI BUDIDAYA JENUH AIR PADA TANAMAN PADI DAN KEDELAI UNTUK ME...PENERAPAN TEKNOLOGI BUDIDAYA JENUH AIR PADA TANAMAN PADI DAN KEDELAI UNTUK ME...
PENERAPAN TEKNOLOGI BUDIDAYA JENUH AIR PADA TANAMAN PADI DAN KEDELAI UNTUK ME...
 
Evaluasi SBO
Evaluasi SBOEvaluasi SBO
Evaluasi SBO
 
Kloning Nukleus
Kloning NukleusKloning Nukleus
Kloning Nukleus
 

Similar to seleksi olimpiade biologi 2012 TINGKAT kabupaten/kota

prediksi soal olimpiade biologi
prediksi soal olimpiade biologiprediksi soal olimpiade biologi
prediksi soal olimpiade biologiLyEnha Cjdw
 
Soal osn-kab-biologi-2012
Soal osn-kab-biologi-2012Soal osn-kab-biologi-2012
Soal osn-kab-biologi-2012LyEnha Cjdw
 
Soal sma biologi_f21
Soal sma biologi_f21Soal sma biologi_f21
Soal sma biologi_f21kiemfull
 
Dokumen ujian nasional biologi 9
Dokumen ujian nasional biologi 9Dokumen ujian nasional biologi 9
Dokumen ujian nasional biologi 9ilhamrevolution2015
 
TRYOUT 4 BIOLOGI TIPE A 2014
TRYOUT 4 BIOLOGI TIPE A 2014TRYOUT 4 BIOLOGI TIPE A 2014
TRYOUT 4 BIOLOGI TIPE A 2014Kasmadi Rais
 
To un 2015 biologi a
To un 2015 biologi aTo un 2015 biologi a
To un 2015 biologi aKasmadi Rais
 
TRYOUT 4 BIOLOGI D 2014
TRYOUT 4 BIOLOGI D 2014TRYOUT 4 BIOLOGI D 2014
TRYOUT 4 BIOLOGI D 2014Kasmadi Rais
 
Soal ON MIPA-PT Biologi 2015
Soal ON MIPA-PT Biologi 2015Soal ON MIPA-PT Biologi 2015
Soal ON MIPA-PT Biologi 2015Nesha Mutiara
 
TRYOUT 4 BIOLOGI C 2014
TRYOUT 4 BIOLOGI C 2014TRYOUT 4 BIOLOGI C 2014
TRYOUT 4 BIOLOGI C 2014Kasmadi Rais
 
Soal bidang-studi-biologi-mts-seleksi-tk-kab-kota-kompetisi-sains-madrasah-ks...
Soal bidang-studi-biologi-mts-seleksi-tk-kab-kota-kompetisi-sains-madrasah-ks...Soal bidang-studi-biologi-mts-seleksi-tk-kab-kota-kompetisi-sains-madrasah-ks...
Soal bidang-studi-biologi-mts-seleksi-tk-kab-kota-kompetisi-sains-madrasah-ks...Muhlis Ahmad
 
Dokumen ujian nasional biologi 4
Dokumen ujian nasional biologi 4Dokumen ujian nasional biologi 4
Dokumen ujian nasional biologi 4ilhamrevolution2015
 
Soal sma biologi_f28
Soal sma biologi_f28Soal sma biologi_f28
Soal sma biologi_f28kiemfull
 
Soal-soal biologi dan pembahasan
Soal-soal biologi dan pembahasanSoal-soal biologi dan pembahasan
Soal-soal biologi dan pembahasanHairlinda Agustin
 
dokumen.tips_download-soal-babak-penyisihan-obi-iv - Copy.docx
dokumen.tips_download-soal-babak-penyisihan-obi-iv - Copy.docxdokumen.tips_download-soal-babak-penyisihan-obi-iv - Copy.docx
dokumen.tips_download-soal-babak-penyisihan-obi-iv - Copy.docxarmy62
 
Dokumen ujian nasional biologi 8
Dokumen ujian nasional biologi 8Dokumen ujian nasional biologi 8
Dokumen ujian nasional biologi 8ilhamrevolution2015
 
Latihan Soal Biologi SMA/MA Menuju UN 2015
Latihan Soal Biologi SMA/MA Menuju UN 2015Latihan Soal Biologi SMA/MA Menuju UN 2015
Latihan Soal Biologi SMA/MA Menuju UN 2015almansyahnis .
 
Prediksisoalunbiologi2013 130404103131-phpapp02
Prediksisoalunbiologi2013 130404103131-phpapp02Prediksisoalunbiologi2013 130404103131-phpapp02
Prediksisoalunbiologi2013 130404103131-phpapp02nasrul awaludin
 
Dokumen ujian nasional biologi 5
Dokumen ujian nasional biologi 5Dokumen ujian nasional biologi 5
Dokumen ujian nasional biologi 5ilhamrevolution2015
 

Similar to seleksi olimpiade biologi 2012 TINGKAT kabupaten/kota (20)

prediksi soal olimpiade biologi
prediksi soal olimpiade biologiprediksi soal olimpiade biologi
prediksi soal olimpiade biologi
 
Soal osn-kab-biologi-2012
Soal osn-kab-biologi-2012Soal osn-kab-biologi-2012
Soal osn-kab-biologi-2012
 
Soal sma biologi_f21
Soal sma biologi_f21Soal sma biologi_f21
Soal sma biologi_f21
 
Dokumen ujian nasional biologi 9
Dokumen ujian nasional biologi 9Dokumen ujian nasional biologi 9
Dokumen ujian nasional biologi 9
 
TRYOUT 4 BIOLOGI TIPE A 2014
TRYOUT 4 BIOLOGI TIPE A 2014TRYOUT 4 BIOLOGI TIPE A 2014
TRYOUT 4 BIOLOGI TIPE A 2014
 
To un 2015 biologi a
To un 2015 biologi aTo un 2015 biologi a
To un 2015 biologi a
 
TRYOUT 4 BIOLOGI D 2014
TRYOUT 4 BIOLOGI D 2014TRYOUT 4 BIOLOGI D 2014
TRYOUT 4 BIOLOGI D 2014
 
Soal ON MIPA-PT Biologi 2015
Soal ON MIPA-PT Biologi 2015Soal ON MIPA-PT Biologi 2015
Soal ON MIPA-PT Biologi 2015
 
TRYOUT 4 BIOLOGI C 2014
TRYOUT 4 BIOLOGI C 2014TRYOUT 4 BIOLOGI C 2014
TRYOUT 4 BIOLOGI C 2014
 
Soal bidang-studi-biologi-mts-seleksi-tk-kab-kota-kompetisi-sains-madrasah-ks...
Soal bidang-studi-biologi-mts-seleksi-tk-kab-kota-kompetisi-sains-madrasah-ks...Soal bidang-studi-biologi-mts-seleksi-tk-kab-kota-kompetisi-sains-madrasah-ks...
Soal bidang-studi-biologi-mts-seleksi-tk-kab-kota-kompetisi-sains-madrasah-ks...
 
Dokumen ujian nasional biologi 4
Dokumen ujian nasional biologi 4Dokumen ujian nasional biologi 4
Dokumen ujian nasional biologi 4
 
Soal sma biologi_f28
Soal sma biologi_f28Soal sma biologi_f28
Soal sma biologi_f28
 
Biologi xii ipa
Biologi xii ipaBiologi xii ipa
Biologi xii ipa
 
Soal-soal biologi dan pembahasan
Soal-soal biologi dan pembahasanSoal-soal biologi dan pembahasan
Soal-soal biologi dan pembahasan
 
dokumen.tips_download-soal-babak-penyisihan-obi-iv - Copy.docx
dokumen.tips_download-soal-babak-penyisihan-obi-iv - Copy.docxdokumen.tips_download-soal-babak-penyisihan-obi-iv - Copy.docx
dokumen.tips_download-soal-babak-penyisihan-obi-iv - Copy.docx
 
Dokumen ujian nasional biologi 8
Dokumen ujian nasional biologi 8Dokumen ujian nasional biologi 8
Dokumen ujian nasional biologi 8
 
Soal biologi
Soal biologiSoal biologi
Soal biologi
 
Latihan Soal Biologi SMA/MA Menuju UN 2015
Latihan Soal Biologi SMA/MA Menuju UN 2015Latihan Soal Biologi SMA/MA Menuju UN 2015
Latihan Soal Biologi SMA/MA Menuju UN 2015
 
Prediksisoalunbiologi2013 130404103131-phpapp02
Prediksisoalunbiologi2013 130404103131-phpapp02Prediksisoalunbiologi2013 130404103131-phpapp02
Prediksisoalunbiologi2013 130404103131-phpapp02
 
Dokumen ujian nasional biologi 5
Dokumen ujian nasional biologi 5Dokumen ujian nasional biologi 5
Dokumen ujian nasional biologi 5
 

More from Moh Hari Rusli

Parabola dan Persamaan garis singgung pada parabola
Parabola dan Persamaan garis singgung pada parabolaParabola dan Persamaan garis singgung pada parabola
Parabola dan Persamaan garis singgung pada parabolaMoh Hari Rusli
 
Fungsi Komposisi dan Fungsi Invers
Fungsi Komposisi dan Fungsi InversFungsi Komposisi dan Fungsi Invers
Fungsi Komposisi dan Fungsi InversMoh Hari Rusli
 
Diktat Pembinaan Olimpiade Matematika Nasional
Diktat Pembinaan Olimpiade Matematika NasionalDiktat Pembinaan Olimpiade Matematika Nasional
Diktat Pembinaan Olimpiade Matematika NasionalMoh Hari Rusli
 
soal Olimpiade Matematika SMA Tingkat Nasional PDIM UB 2014
soal Olimpiade Matematika SMA Tingkat Nasional PDIM UB 2014soal Olimpiade Matematika SMA Tingkat Nasional PDIM UB 2014
soal Olimpiade Matematika SMA Tingkat Nasional PDIM UB 2014Moh Hari Rusli
 
Soal Olimpiade Matematika SMA Tingkat Nasional PDIM UB 2013
Soal Olimpiade Matematika SMA Tingkat Nasional PDIM UB 2013Soal Olimpiade Matematika SMA Tingkat Nasional PDIM UB 2013
Soal Olimpiade Matematika SMA Tingkat Nasional PDIM UB 2013Moh Hari Rusli
 
Kunci jawaban Olimpiade Matematika SMA Tingkat Nasional PDIM UNIVERSITAS BRAW...
Kunci jawaban Olimpiade Matematika SMA Tingkat Nasional PDIM UNIVERSITAS BRAW...Kunci jawaban Olimpiade Matematika SMA Tingkat Nasional PDIM UNIVERSITAS BRAW...
Kunci jawaban Olimpiade Matematika SMA Tingkat Nasional PDIM UNIVERSITAS BRAW...Moh Hari Rusli
 
Soal Olimpiade Matematika SMA Tingkat Nasional PDIM UB 2012
Soal Olimpiade Matematika SMA Tingkat Nasional PDIM UB 2012Soal Olimpiade Matematika SMA Tingkat Nasional PDIM UB 2012
Soal Olimpiade Matematika SMA Tingkat Nasional PDIM UB 2012Moh Hari Rusli
 
Tradisi tujuh hari dalam Islam
Tradisi tujuh hari dalam IslamTradisi tujuh hari dalam Islam
Tradisi tujuh hari dalam IslamMoh Hari Rusli
 
Konsep, Hukum, dan Dalil Membaca al qur’an di kuburan
Konsep, Hukum, dan Dalil Membaca al qur’an di kuburanKonsep, Hukum, dan Dalil Membaca al qur’an di kuburan
Konsep, Hukum, dan Dalil Membaca al qur’an di kuburanMoh Hari Rusli
 
Kosep dan Dalil Maulid Nabi Muhammad SAW
Kosep dan Dalil Maulid Nabi Muhammad SAWKosep dan Dalil Maulid Nabi Muhammad SAW
Kosep dan Dalil Maulid Nabi Muhammad SAWMoh Hari Rusli
 
Konsep Kehidupan alam barzakh
Konsep Kehidupan alam barzakhKonsep Kehidupan alam barzakh
Konsep Kehidupan alam barzakhMoh Hari Rusli
 
Konsep dan Dalil tabarruk
Konsep dan Dalil tabarrukKonsep dan Dalil tabarruk
Konsep dan Dalil tabarrukMoh Hari Rusli
 
Konsep dan Dalil Tawasul dan istighatsah
Konsep dan Dalil Tawasul dan istighatsahKonsep dan Dalil Tawasul dan istighatsah
Konsep dan Dalil Tawasul dan istighatsahMoh Hari Rusli
 
Konsep dan Dalil Hadiah pahala al qur’an
Konsep dan Dalil Hadiah pahala al qur’anKonsep dan Dalil Hadiah pahala al qur’an
Konsep dan Dalil Hadiah pahala al qur’anMoh Hari Rusli
 
Konsep dan Dalil Yasin fadhilah
Konsep dan Dalil Yasin fadhilahKonsep dan Dalil Yasin fadhilah
Konsep dan Dalil Yasin fadhilahMoh Hari Rusli
 
KEAJAIBAN DIALOG AL-QUR’AN DENGAN PERKEMBANGAN ILMU PENGETAHUAN
KEAJAIBAN DIALOG AL-QUR’AN DENGAN PERKEMBANGAN ILMU PENGETAHUANKEAJAIBAN DIALOG AL-QUR’AN DENGAN PERKEMBANGAN ILMU PENGETAHUAN
KEAJAIBAN DIALOG AL-QUR’AN DENGAN PERKEMBANGAN ILMU PENGETAHUANMoh Hari Rusli
 
Konsep dan Dalil tentang bid'ah hasanah
Konsep dan Dalil tentang bid'ah hasanahKonsep dan Dalil tentang bid'ah hasanah
Konsep dan Dalil tentang bid'ah hasanahMoh Hari Rusli
 
BSE Pendidikan Kewarganegaraan SMP/MTs Kelas 9
BSE Pendidikan Kewarganegaraan SMP/MTs Kelas 9BSE Pendidikan Kewarganegaraan SMP/MTs Kelas 9
BSE Pendidikan Kewarganegaraan SMP/MTs Kelas 9Moh Hari Rusli
 

More from Moh Hari Rusli (20)

Irisan kerucut
Irisan kerucutIrisan kerucut
Irisan kerucut
 
Parabola dan Persamaan garis singgung pada parabola
Parabola dan Persamaan garis singgung pada parabolaParabola dan Persamaan garis singgung pada parabola
Parabola dan Persamaan garis singgung pada parabola
 
Fungsi Komposisi dan Fungsi Invers
Fungsi Komposisi dan Fungsi InversFungsi Komposisi dan Fungsi Invers
Fungsi Komposisi dan Fungsi Invers
 
Diktat Pembinaan Olimpiade Matematika Nasional
Diktat Pembinaan Olimpiade Matematika NasionalDiktat Pembinaan Olimpiade Matematika Nasional
Diktat Pembinaan Olimpiade Matematika Nasional
 
soal Olimpiade Matematika SMA Tingkat Nasional PDIM UB 2014
soal Olimpiade Matematika SMA Tingkat Nasional PDIM UB 2014soal Olimpiade Matematika SMA Tingkat Nasional PDIM UB 2014
soal Olimpiade Matematika SMA Tingkat Nasional PDIM UB 2014
 
Soal Olimpiade Matematika SMA Tingkat Nasional PDIM UB 2013
Soal Olimpiade Matematika SMA Tingkat Nasional PDIM UB 2013Soal Olimpiade Matematika SMA Tingkat Nasional PDIM UB 2013
Soal Olimpiade Matematika SMA Tingkat Nasional PDIM UB 2013
 
Kunci jawaban Olimpiade Matematika SMA Tingkat Nasional PDIM UNIVERSITAS BRAW...
Kunci jawaban Olimpiade Matematika SMA Tingkat Nasional PDIM UNIVERSITAS BRAW...Kunci jawaban Olimpiade Matematika SMA Tingkat Nasional PDIM UNIVERSITAS BRAW...
Kunci jawaban Olimpiade Matematika SMA Tingkat Nasional PDIM UNIVERSITAS BRAW...
 
Soal Olimpiade Matematika SMA Tingkat Nasional PDIM UB 2012
Soal Olimpiade Matematika SMA Tingkat Nasional PDIM UB 2012Soal Olimpiade Matematika SMA Tingkat Nasional PDIM UB 2012
Soal Olimpiade Matematika SMA Tingkat Nasional PDIM UB 2012
 
Tradisi tujuh hari dalam Islam
Tradisi tujuh hari dalam IslamTradisi tujuh hari dalam Islam
Tradisi tujuh hari dalam Islam
 
Konsep, Hukum, dan Dalil Membaca al qur’an di kuburan
Konsep, Hukum, dan Dalil Membaca al qur’an di kuburanKonsep, Hukum, dan Dalil Membaca al qur’an di kuburan
Konsep, Hukum, dan Dalil Membaca al qur’an di kuburan
 
Kosep dan Dalil Maulid Nabi Muhammad SAW
Kosep dan Dalil Maulid Nabi Muhammad SAWKosep dan Dalil Maulid Nabi Muhammad SAW
Kosep dan Dalil Maulid Nabi Muhammad SAW
 
Konsep Kehidupan alam barzakh
Konsep Kehidupan alam barzakhKonsep Kehidupan alam barzakh
Konsep Kehidupan alam barzakh
 
ISLAM DAN TRADISI
ISLAM DAN TRADISIISLAM DAN TRADISI
ISLAM DAN TRADISI
 
Konsep dan Dalil tabarruk
Konsep dan Dalil tabarrukKonsep dan Dalil tabarruk
Konsep dan Dalil tabarruk
 
Konsep dan Dalil Tawasul dan istighatsah
Konsep dan Dalil Tawasul dan istighatsahKonsep dan Dalil Tawasul dan istighatsah
Konsep dan Dalil Tawasul dan istighatsah
 
Konsep dan Dalil Hadiah pahala al qur’an
Konsep dan Dalil Hadiah pahala al qur’anKonsep dan Dalil Hadiah pahala al qur’an
Konsep dan Dalil Hadiah pahala al qur’an
 
Konsep dan Dalil Yasin fadhilah
Konsep dan Dalil Yasin fadhilahKonsep dan Dalil Yasin fadhilah
Konsep dan Dalil Yasin fadhilah
 
KEAJAIBAN DIALOG AL-QUR’AN DENGAN PERKEMBANGAN ILMU PENGETAHUAN
KEAJAIBAN DIALOG AL-QUR’AN DENGAN PERKEMBANGAN ILMU PENGETAHUANKEAJAIBAN DIALOG AL-QUR’AN DENGAN PERKEMBANGAN ILMU PENGETAHUAN
KEAJAIBAN DIALOG AL-QUR’AN DENGAN PERKEMBANGAN ILMU PENGETAHUAN
 
Konsep dan Dalil tentang bid'ah hasanah
Konsep dan Dalil tentang bid'ah hasanahKonsep dan Dalil tentang bid'ah hasanah
Konsep dan Dalil tentang bid'ah hasanah
 
BSE Pendidikan Kewarganegaraan SMP/MTs Kelas 9
BSE Pendidikan Kewarganegaraan SMP/MTs Kelas 9BSE Pendidikan Kewarganegaraan SMP/MTs Kelas 9
BSE Pendidikan Kewarganegaraan SMP/MTs Kelas 9
 

Recently uploaded

Estetika Humanisme Diskusi Video Sesi Ke-1.pdf
Estetika Humanisme Diskusi Video Sesi Ke-1.pdfEstetika Humanisme Diskusi Video Sesi Ke-1.pdf
Estetika Humanisme Diskusi Video Sesi Ke-1.pdfHendroGunawan8
 
AKSI NYATA MODUL 1.3 VISI GURU PENGGERAK.pptx
AKSI NYATA MODUL 1.3 VISI GURU PENGGERAK.pptxAKSI NYATA MODUL 1.3 VISI GURU PENGGERAK.pptx
AKSI NYATA MODUL 1.3 VISI GURU PENGGERAK.pptxHeriyantoHeriyanto44
 
(NEW) Template Presentasi UGM yang terbaru
(NEW) Template Presentasi UGM yang terbaru(NEW) Template Presentasi UGM yang terbaru
(NEW) Template Presentasi UGM yang terbaruSilvanaAyu
 
slide presentation bab 2 sain form 2.pdf
slide presentation bab 2 sain form 2.pdfslide presentation bab 2 sain form 2.pdf
slide presentation bab 2 sain form 2.pdfNURAFIFAHBINTIJAMALU
 
Silabus Mata Pelajaran Biologi SMA Kelas X.doc
Silabus Mata Pelajaran Biologi SMA Kelas X.docSilabus Mata Pelajaran Biologi SMA Kelas X.doc
Silabus Mata Pelajaran Biologi SMA Kelas X.docNurulAiniFirdasari1
 
rpp bangun-ruang-sisi-datar kelas 8 smp.pdf
rpp bangun-ruang-sisi-datar kelas 8 smp.pdfrpp bangun-ruang-sisi-datar kelas 8 smp.pdf
rpp bangun-ruang-sisi-datar kelas 8 smp.pdfGugunGunawan93
 
Materi Kelas Online Ministry Learning Center - Bedah Kitab 1 Tesalonika
Materi Kelas Online Ministry Learning Center - Bedah Kitab 1 TesalonikaMateri Kelas Online Ministry Learning Center - Bedah Kitab 1 Tesalonika
Materi Kelas Online Ministry Learning Center - Bedah Kitab 1 TesalonikaSABDA
 
AKSI NYATA MODUL 1.3 VISI GURU PENGGERAK.pdf
AKSI NYATA MODUL 1.3 VISI GURU PENGGERAK.pdfAKSI NYATA MODUL 1.3 VISI GURU PENGGERAK.pdf
AKSI NYATA MODUL 1.3 VISI GURU PENGGERAK.pdfHeriyantoHeriyanto44
 
Gandum & Lalang (Matius......13_24-30).pptx
Gandum & Lalang (Matius......13_24-30).pptxGandum & Lalang (Matius......13_24-30).pptx
Gandum & Lalang (Matius......13_24-30).pptxHansTobing
 
PLaN & INTERVENSI untuk sekolah yang memerlukan
PLaN & INTERVENSI untuk sekolah yang memerlukanPLaN & INTERVENSI untuk sekolah yang memerlukan
PLaN & INTERVENSI untuk sekolah yang memerlukanssuserc81826
 
PAMPHLET PENGAKAP aktiviti pengakap 2024
PAMPHLET PENGAKAP aktiviti pengakap 2024PAMPHLET PENGAKAP aktiviti pengakap 2024
PAMPHLET PENGAKAP aktiviti pengakap 2024MALISAAININOORBINTIA
 
5. HAK DAN KEWAJIBAN JEMAAH indonesia.pdf
5. HAK DAN KEWAJIBAN JEMAAH indonesia.pdf5. HAK DAN KEWAJIBAN JEMAAH indonesia.pdf
5. HAK DAN KEWAJIBAN JEMAAH indonesia.pdfWahyudinST
 
Aksi Nyata PERENCANAAN BERBASIS DATA.pptx
Aksi Nyata PERENCANAAN BERBASIS DATA.pptxAksi Nyata PERENCANAAN BERBASIS DATA.pptx
Aksi Nyata PERENCANAAN BERBASIS DATA.pptxdonny761155
 
Pembuktian rumus volume dan luas permukaan bangung ruang Tabung, Limas, Keruc...
Pembuktian rumus volume dan luas permukaan bangung ruang Tabung, Limas, Keruc...Pembuktian rumus volume dan luas permukaan bangung ruang Tabung, Limas, Keruc...
Pembuktian rumus volume dan luas permukaan bangung ruang Tabung, Limas, Keruc...NiswatuzZahroh
 
UNSUR - UNSUR, LUAS, KELILING LINGKARAN.pptx
UNSUR - UNSUR, LUAS, KELILING LINGKARAN.pptxUNSUR - UNSUR, LUAS, KELILING LINGKARAN.pptx
UNSUR - UNSUR, LUAS, KELILING LINGKARAN.pptxFranxisca Kurniawati
 
PPT uji anova keterangan dan contoh soal.ppt
PPT uji anova keterangan dan contoh soal.pptPPT uji anova keterangan dan contoh soal.ppt
PPT uji anova keterangan dan contoh soal.pptBennyKurniawan42
 
SKPM Kualiti @ Sekolah 23 Feb 22222023.pptx
SKPM Kualiti @ Sekolah 23 Feb 22222023.pptxSKPM Kualiti @ Sekolah 23 Feb 22222023.pptx
SKPM Kualiti @ Sekolah 23 Feb 22222023.pptxg66527130
 
Diagram Fryer Pembelajaran Berdifferensiasi
Diagram Fryer Pembelajaran BerdifferensiasiDiagram Fryer Pembelajaran Berdifferensiasi
Diagram Fryer Pembelajaran BerdifferensiasiOviLarassaty1
 
KISI-KISI Soal PAS Geografi Kelas XII.docx
KISI-KISI Soal PAS Geografi Kelas XII.docxKISI-KISI Soal PAS Geografi Kelas XII.docx
KISI-KISI Soal PAS Geografi Kelas XII.docxjohan effendi
 
Keberagaman-Peserta-Didik-dalam-Psikologi-Pendidikan.pptx
Keberagaman-Peserta-Didik-dalam-Psikologi-Pendidikan.pptxKeberagaman-Peserta-Didik-dalam-Psikologi-Pendidikan.pptx
Keberagaman-Peserta-Didik-dalam-Psikologi-Pendidikan.pptxLeniMawarti1
 

Recently uploaded (20)

Estetika Humanisme Diskusi Video Sesi Ke-1.pdf
Estetika Humanisme Diskusi Video Sesi Ke-1.pdfEstetika Humanisme Diskusi Video Sesi Ke-1.pdf
Estetika Humanisme Diskusi Video Sesi Ke-1.pdf
 
AKSI NYATA MODUL 1.3 VISI GURU PENGGERAK.pptx
AKSI NYATA MODUL 1.3 VISI GURU PENGGERAK.pptxAKSI NYATA MODUL 1.3 VISI GURU PENGGERAK.pptx
AKSI NYATA MODUL 1.3 VISI GURU PENGGERAK.pptx
 
(NEW) Template Presentasi UGM yang terbaru
(NEW) Template Presentasi UGM yang terbaru(NEW) Template Presentasi UGM yang terbaru
(NEW) Template Presentasi UGM yang terbaru
 
slide presentation bab 2 sain form 2.pdf
slide presentation bab 2 sain form 2.pdfslide presentation bab 2 sain form 2.pdf
slide presentation bab 2 sain form 2.pdf
 
Silabus Mata Pelajaran Biologi SMA Kelas X.doc
Silabus Mata Pelajaran Biologi SMA Kelas X.docSilabus Mata Pelajaran Biologi SMA Kelas X.doc
Silabus Mata Pelajaran Biologi SMA Kelas X.doc
 
rpp bangun-ruang-sisi-datar kelas 8 smp.pdf
rpp bangun-ruang-sisi-datar kelas 8 smp.pdfrpp bangun-ruang-sisi-datar kelas 8 smp.pdf
rpp bangun-ruang-sisi-datar kelas 8 smp.pdf
 
Materi Kelas Online Ministry Learning Center - Bedah Kitab 1 Tesalonika
Materi Kelas Online Ministry Learning Center - Bedah Kitab 1 TesalonikaMateri Kelas Online Ministry Learning Center - Bedah Kitab 1 Tesalonika
Materi Kelas Online Ministry Learning Center - Bedah Kitab 1 Tesalonika
 
AKSI NYATA MODUL 1.3 VISI GURU PENGGERAK.pdf
AKSI NYATA MODUL 1.3 VISI GURU PENGGERAK.pdfAKSI NYATA MODUL 1.3 VISI GURU PENGGERAK.pdf
AKSI NYATA MODUL 1.3 VISI GURU PENGGERAK.pdf
 
Gandum & Lalang (Matius......13_24-30).pptx
Gandum & Lalang (Matius......13_24-30).pptxGandum & Lalang (Matius......13_24-30).pptx
Gandum & Lalang (Matius......13_24-30).pptx
 
PLaN & INTERVENSI untuk sekolah yang memerlukan
PLaN & INTERVENSI untuk sekolah yang memerlukanPLaN & INTERVENSI untuk sekolah yang memerlukan
PLaN & INTERVENSI untuk sekolah yang memerlukan
 
PAMPHLET PENGAKAP aktiviti pengakap 2024
PAMPHLET PENGAKAP aktiviti pengakap 2024PAMPHLET PENGAKAP aktiviti pengakap 2024
PAMPHLET PENGAKAP aktiviti pengakap 2024
 
5. HAK DAN KEWAJIBAN JEMAAH indonesia.pdf
5. HAK DAN KEWAJIBAN JEMAAH indonesia.pdf5. HAK DAN KEWAJIBAN JEMAAH indonesia.pdf
5. HAK DAN KEWAJIBAN JEMAAH indonesia.pdf
 
Aksi Nyata PERENCANAAN BERBASIS DATA.pptx
Aksi Nyata PERENCANAAN BERBASIS DATA.pptxAksi Nyata PERENCANAAN BERBASIS DATA.pptx
Aksi Nyata PERENCANAAN BERBASIS DATA.pptx
 
Pembuktian rumus volume dan luas permukaan bangung ruang Tabung, Limas, Keruc...
Pembuktian rumus volume dan luas permukaan bangung ruang Tabung, Limas, Keruc...Pembuktian rumus volume dan luas permukaan bangung ruang Tabung, Limas, Keruc...
Pembuktian rumus volume dan luas permukaan bangung ruang Tabung, Limas, Keruc...
 
UNSUR - UNSUR, LUAS, KELILING LINGKARAN.pptx
UNSUR - UNSUR, LUAS, KELILING LINGKARAN.pptxUNSUR - UNSUR, LUAS, KELILING LINGKARAN.pptx
UNSUR - UNSUR, LUAS, KELILING LINGKARAN.pptx
 
PPT uji anova keterangan dan contoh soal.ppt
PPT uji anova keterangan dan contoh soal.pptPPT uji anova keterangan dan contoh soal.ppt
PPT uji anova keterangan dan contoh soal.ppt
 
SKPM Kualiti @ Sekolah 23 Feb 22222023.pptx
SKPM Kualiti @ Sekolah 23 Feb 22222023.pptxSKPM Kualiti @ Sekolah 23 Feb 22222023.pptx
SKPM Kualiti @ Sekolah 23 Feb 22222023.pptx
 
Diagram Fryer Pembelajaran Berdifferensiasi
Diagram Fryer Pembelajaran BerdifferensiasiDiagram Fryer Pembelajaran Berdifferensiasi
Diagram Fryer Pembelajaran Berdifferensiasi
 
KISI-KISI Soal PAS Geografi Kelas XII.docx
KISI-KISI Soal PAS Geografi Kelas XII.docxKISI-KISI Soal PAS Geografi Kelas XII.docx
KISI-KISI Soal PAS Geografi Kelas XII.docx
 
Keberagaman-Peserta-Didik-dalam-Psikologi-Pendidikan.pptx
Keberagaman-Peserta-Didik-dalam-Psikologi-Pendidikan.pptxKeberagaman-Peserta-Didik-dalam-Psikologi-Pendidikan.pptx
Keberagaman-Peserta-Didik-dalam-Psikologi-Pendidikan.pptx
 

seleksi olimpiade biologi 2012 TINGKAT kabupaten/kota

  • 1. KEMENTERIAN PENDIDIKAN NASIONAL DIREKTORAT JENDERAL PENDIDIKAN MENENGAH DIREKTORAT PEMBINAAN SEKOLAH MENENGAH ATAS TEST TINGKAT KABUPATEN/KOTA SELEKSI CALON PESERTA INTERNATIONAL BIOLOGY OLYMPIAD (IBO) TAHUN 2012 KODE: 03 PETUNJUK: 1. Setiap soal memiliki bobot nilai 1. 2. Isilah nama, asal sekolah, kelas, dan kode soal pada lembar jawaban. 3. Gunakan ballpoint/pulpen untuk mengisi jawaban yang tepat pada lembar yang telah disediakan dengan cara mencantumkan hurufnya saja. 4. Tidak ada sistem minus. 5. Waktu Test = 120 menit. TIM OLIMPIADE BIOLOGI INDONESIA 2011 03-OSK-2011 Copyright © 2011 by Kemdiknas Hak Cipta dilindungi oleh Undang-undang 0
  • 2. Nama: ......................................................................................................... Asal SMA/Kelas: ........................................................................................ Seleksi Calon Peserta IBO 2012 Tingkat Kabupaten/Kota (Kode:03) BIOLOGI SEL DAN MOLEKULER (Nilai 20) 1. Manakah pasangan berikut ini yang TIDAK merepresentasikan hubungan yang benar? A. Lemak; lipid B. Pati; polisakarida C. Pati; karbohidrat D. Gula; karbohidrat E. Enzim; lipid 2. Di dalam sel hewan, cadangan makanan berupa lipid disimpan dalam bentuk molekul: A. asam lemak B. trigliserida C. glikogen D. lipid bilayer E. badan keton 3. Perhatikan gambar grafik berikut ini. Berdasarkan grafik di atas, berapa persentase perubahan pada rasio luas permukaan-volume apabila sebuah sel berkembang dari diameter 1 m menjadi 2 m? A. 10% B. 20% C. 30% D. 50% E. 90% 4. Berdasarkan grafik pada pertanyaan No.3 di atas, berapakah diameter maksimum yang dapat ditunjang oleh sel tersebut sebelum rasio luas permukaan - volume mencapai nilai di bawah 1? A. 2 m B. 5 m C. 15 m D. 10 m E. 20 m 1 03-OSK-2011 Copyright © 2011 by Kemdiknas Hak Cipta dilindungi oleh Undang-undang
  • 3. Nama: ......................................................................................................... Asal SMA/Kelas: ........................................................................................ Seleksi Calon Peserta IBO 2012 Tingkat Kabupaten/Kota (Kode:03) 5. Sel kelenjar yang mampu memproduksi hormon insulin dalam jumlah besar umumnya memiliki organel (.....) yang berkembang dengan baik. A. silia B. sentriol C. RE kasar D. RE halus E. lisosom 6. Sel makrofag pada sistem imun berperan dalam menelan bakteri atau zat asing lainnya. Organel yang berperan penting dalam proses ini adalah: A. sentriol B. nukleus C. peroksisom D. lisosom E. RE halus 7. Menurut anda, apakah yang akan terjadi apabila lisosom pada sel pecah dan mengeluarkan isinya ke dalam sitoplasma? A. Peningkatan produksi protein B. Terjadinya pembelahan mitokondria dan kloroplas C. Makromolekul di dalam sitosol terdegradasi D. DNA di dalam mitokondria terdegradasi E. DNA di dalam kloroplas terdegradasi 8. Manakah dari pernyataan mengenai struktur dan organel sel berikut yang benar? A. Pada sel hewan dan tumbuhan, retikulum endoplasma terhubung dengan membran luar dari nukleus B. Pada umumnya, nukleus merupakan organel dengan ukuran terbesar pada tumbuhan multiseluler C. Sambungan celah (gap junction) dan plasmodesmata pada dasarnya adalah struktur yang sama, namun diberi nama yang berbeda pada sel hewan dan tumbuhan D. Sel tumbuhan berbeda dengan sel hewan karena mereka tidak memiliki mitokondria E. Sel tumbuhan berbeda dengan sel hewan karena mereka tidak memiliki sitoskeleton 9. Plastida yang berfungsi untuk menyimpan lemak pada sel tumbuhan disebut: A. kloroplas B. amiloplas C. leukoplas D. kromoplas E. peroksisom 10. Ketika sel-sel epitel silindris pada saluran pencernaan diberikan suatu senyawa kimia X, bentuknya berubah menjadi bulat melingkar. Struktur internal sel yang terganggu oleh senyawa kimia X tersebut adalah: A. mitokondria B. membran sel C. retikulum endoplasma D. mikrotubul E. sambungan sel (cell junctions) 2 03-OSK-2011 Copyright © 2011 by Kemdiknas Hak Cipta dilindungi oleh Undang-undang
  • 4. Nama: ......................................................................................................... Asal SMA/Kelas: ........................................................................................ Seleksi Calon Peserta IBO 2012 Tingkat Kabupaten/Kota (Kode:03) 11. Dengan memperhatikan peran Ca2+ (kalsium) dalam polimerisasi aktin, maka penurunan secara signifikan jumlah Ca2+ ekstraseluler pada medium Euglena dapat menyebabkan: A. sel bergerak cepat B. sel cenderung membulat dan memendek C. peningkatan volume sitoplasma D. pemanjangan membran sel E. menghambat fluiditas membran 12. Molekul air dapat melewati membran sel melalui: A. protein perifer B. aquaporin C. protein membran dan menggunakan ATP D. sistem kotransport E. endositosis 13. Tipe golongan darah A, B, dan O dapat ditandai melalui molekul (.....) pada permukaan sel A. glikoprotein B. glikolipid C. lipoprotein D. gliserol E. glutamin 14. Sel-sel di dalam tubuh suatu individu dapat mengenal satu sama lain melalui komponen (.....) pada membran sel A. glikolipid B. posfat C. glikoprotein D. lipoprotein E. ketoglutarat 15. Apa yang akan terjadi jika sel darah merah manusia ditempatkan pada air distilasi? A. Sel akan mengkerut B. Sel akan mengalami plasmolisis C. Sel akan pecah D. Sel akan kehilangan air E. Sel tidak mengalami perubahan 16. Threonin (asam amino) dan arabinosa (monosakarida) dapat masuk ke dalam sel melalui: A. difusi B. endocytosis C. fosforilasi D. difusi terfasilitasi E. osmosis 17. Proses apakah yang dijelaskan oleh mekanisme kemiosmosis? A. Pergerakan molekul air ke dalam kloroplas dan penggunaannya dalam fotosintesis B. Pembentukan ATP yang dipasangkan dengan aliran proton (H+) pada respirasi aerob C. Pergerakan molekul hidrofobik melintasi membran sel D. Pergerakan molekul NADH dari sitoplasma ke dalam mitokondria E. Pembentukan gradien kimiawi oleh osmosis 3 03-OSK-2011 Copyright © 2011 by Kemdiknas Hak Cipta dilindungi oleh Undang-undang
  • 5. Nama: ......................................................................................................... Asal SMA/Kelas: ........................................................................................ Seleksi Calon Peserta IBO 2012 Tingkat Kabupaten/Kota (Kode:03) 18. Drosophila melanogaster memiliki empat pasang kromosom. Jika sperma dari spesies ini terbentuk melalui proses meiosis tanpa mengalami crossing over, berapa banyak kombinasi genetik pada sel sperma dapat diproduksi oleh jantan Drosophila melanogaster? A. 4 B. 8 C. 16 D. 64 E. 256 19. Diagram di bawah menunjukkan perubahan jumlah DNA di dalam sel (ordinat/sumbu Y) sebagai fungsi dari waktu (absis/sumbu X) selama siklus sel. Diagram manakah yang menunjukkan perubahan jumlah DNA di dalam sel pada waktu meiosis dan mitosis? A. B. C. D. E. Meiosis = 2, Mitosis = 4 Meiosis = 3, Mitosis = 1 Meiosis = 3, Mitosis = 2 Meiosis = 4, Mitosis = 1 Meiosis = 4, Mitosis = 2 20. Salah satu cara penanggulangan influenza adalah melalui vaksinasi, namun hampir setiap tahun harus dibuat jenis vaksin flu yang baru. Hal ini terutama disebabkan karena: A. virus yang berbeda menyerang penduduk pada usia yang berbeda sehingga setiap tahun harus diproduksi vaksin yang baru untuk kelompok usia berbeda B. antibodi yang diproduksi tubuh terhadap vaksin flu tidak bertahan lama dalam darah C. vaksin umumnya bersifat tidak stabil sehingga tidak dapat disimpan lebih dari setahun D. tubuh kita belajar untuk menghancurkan antibodi yang telah diproduksi terhadap vaksin sehingga diperlukan jenis vaksin baru untuk setiap kali vaksinasi E. virus influenza memiliki frekuensi mutasi yang tinggi 4 03-OSK-2011 Copyright © 2011 by Kemdiknas Hak Cipta dilindungi oleh Undang-undang
  • 6. Nama: ......................................................................................................... Asal SMA/Kelas: ........................................................................................ Seleksi Calon Peserta IBO 2012 Tingkat Kabupaten/Kota (Kode:03) ANATOMI DAN FISIOLOGI TUMBUHAN (Nilai 15) 1. Selama fotosintesis, karbohidrat dibuat oleh tumbuhan: A. selama reaksi terang B. dari oksigen, air, dan tiga molekul karbondioksida C. di grana D. di stroma E. agar dapat memulai reaksi terang 2. Urutan yang tepat menggambarkan aliran elektron yang terjadi pada waktu fotosintesis adalah: A. air – klorofil – NADP+ B. air – NADP+ – klorofil C. klorofil – air – NADP+ D. klorofil – NADP+ – air E. air – NADP+ – klorofil 3. Jika seorang ilmuwan menambahkan senyawa kimia yang khusus menghambat difusi proton yang keluar dari tilakoid melalui kompleks ATP sintase, hasil apa yang akan diperoleh? A. meningkatkan NADP+ pada bagian dalam tilakoid B. meningkatkan pH di bagian dalam tilakoid C. produksi ATP yang meningkat D. tidak ada perubahan dalam produksi ATP E. produksi ATP menurun 4. Spektrum penyerapan cahaya pada klorofil a tidak identik dengan spektrum fotosintesis karena: A. terdapat pigmen aksesori yang turut berkontribusi pada proses fotosintesis B. klorofil a menyerap baik cahaya merah maupun biru C. klorofil a memantulkan cahaya hijau D. perbedaan panjang gelombang cahaya menghasilkan perbedaan energi yang dihasilkan E. klorofil a dapat diaktifkan dengan menyerap foton dari cahaya 5. Fotosintesis terjadi pada: A. sel-sel sklerenkim B. sel-sel kolenkim C. sel-sel penjaga D. sel-sel parenkim E. jawaban C dan D 6. Efek apa yang terjadi pada tumbuhan ketika diberi larutan 10% sukrosa? A. stomata akan membuka sehingga menyebabkan tumbuhan kehilangan air B. stomata akan menutup sehingga tumbuhan kekurangan karbondioksida C. stomata akan menutup sehinggah sehingga tumbuhan kekurangan oksigen D. sel tumbuhan akan terisi air dari osmosis sehingga tumbuhan menjadi turgid E. jawaban B dan C 5 03-OSK-2011 Copyright © 2011 by Kemdiknas Hak Cipta dilindungi oleh Undang-undang
  • 7. Nama: ......................................................................................................... Asal SMA/Kelas: ........................................................................................ 7. Seleksi Calon Peserta IBO 2012 Tingkat Kabupaten/Kota (Kode:03) Perhatikan gambar berikut ini. Tumbuhan yang memiliki anatomi organ seperti gambar di atas memiliki kemampuan untuk: A. melakukan fotosintesis di mesofil dan seludang pembuluh B. menghasilkan asam oksalat dari CO2 C. melakukan fotorespirasi D. jawaban B dan C E. jawaban A dan B 8. Tekanan akar pada tumbuhan akan menyebabkan air bergerak pada sel-sel akar melalui proses: A. translokasi B. adhesi C. osmosis D. kerja kapiler E. gutasi 9. Perhatikan gambar dibawah ini. Pernyataan yang tepat untuk menjelaskan gambar di atas adalah: A. nirogen dapat langsung diikat (difiksasi akar tumbuhan) B. nitrogen dapat diserap langsung oleh stomata C. nitrogen mengalami perubahan menjadi asam amino oleh aktivitas bakteria D. nitrogen dapat kembali ditranspirasikan ke atmosfer melalui stomata E. nitrogen mengalami amonifikasi dengan bantuan bakteri 6 03-OSK-2011 Copyright © 2011 by Kemdiknas Hak Cipta dilindungi oleh Undang-undang
  • 8. Nama: ......................................................................................................... Asal SMA/Kelas: ........................................................................................ Seleksi Calon Peserta IBO 2012 Tingkat Kabupaten/Kota (Kode:03) 10. Tumbuhan menjadi kerdil dan mengalami klorosis diakibatkan terjadi defisiensi unsur berikut secara berturut-turut, yaitu: A. N-P dan Mg-Ca B. N-Mg dan Ca-P C. Mg-P dan N-Ca D. Mg-Ca dan N-P E. P-Ca dan Mg-N 11. Selama proses reproduksi pada tumbuhan berbunga, terjadi beberapa perubahan sebagai berikut, KECUALI: A. mikrospora berubah menjadi serbuk polen B. ovarium menjadi buah C. ovulum menjadi biji D. kelopak bunga berguguran E. tabung polen menjadi inti sperma 12. Selama proses reproduksi tumbuhan, semua transisi berikut ini terjadi, KECUALI: A. mikrospora menjadi butir pollen B. ovarium menjadi buah C. ovulum menjadi biji D. mahkota bunga gugur E. inti tabung menjadi inti sperma 13. Perhatikan gambar sayatan longitudinal dari ovul tumbuhan angiospermae. Dengan struktur pada nomor berapakah, isi dari tabung polen akan berfusi? A. 1 dan 3 B. 1 dan 4 C. 2 dan 3 D. 3 dan 4 E. 2 dan 4 7 03-OSK-2011 Copyright © 2011 by Kemdiknas Hak Cipta dilindungi oleh Undang-undang
  • 9. Nama: ......................................................................................................... Asal SMA/Kelas: ........................................................................................ Seleksi Calon Peserta IBO 2012 Tingkat Kabupaten/Kota (Kode:03) 14. Peristiwa berikut yang TIDAK berkaitan dengan peran auksin pada tumbuhan adalah: A. dominansi apikal B. partenokarpi C. mencegah absisi D. memecah masa istirahat biji (breaking dormancy) E. memicu pertumbuhan akar adventif 15. Berapa tahun usia batang pohon yang tumbuh di daerah temperata dengan hasil sayatan melintang seperti gambar di bawah ini? A. B. C. D. E. 4 tahun 6 tahun 8 tahun 10 tahun 14 tahun 8 03-OSK-2011 Copyright © 2011 by Kemdiknas Hak Cipta dilindungi oleh Undang-undang
  • 10. Nama: ......................................................................................................... Asal SMA/Kelas: ........................................................................................ Seleksi Calon Peserta IBO 2012 Tingkat Kabupaten/Kota (Kode:03) ANATOMI DAN FISIOLOGI HEWAN (Nilai 25) 1. Cairan ekstraseluler yang mengelilingi sel-sel penyusun tubuh hewan berasal dari: A. plasma darah B. kelenjar eksokrin C. kelenjar endokrin D. jaringan limfatik E. hasil filtrasi ginjal 2. Manakah material berikut ini yang TIDAK dapat dicerna oleh sapi? A. Protein B. Trigliserida C. Glikogen D. Selulosa E. Asam nukleat 3. Pernyataan di bawah ini yang berlaku untuk seluruh steroid adalah: A. Seluruh steroid larut di dalam seluruh vitamin B. Sel target dari setiap steroid adalah setiap sel di dalam tubuh C. Steroid berikatan kepada reseptor protein yang terdapat pada membran dari sel target mereka D. Steroid disintesis pada reticulum endoplasma kasar E. Steroid larut di dalam senyawa lipid 4. Pankreas adalah organ tubuh yang unik karena memiliki fungsi eksokrin dan endokrin. Fungsi eksokrin dari pankreas adalah melepaskan: A. enzim pencernaan ke darah B. enzim pencernaan melalui suatu saluran C. hormon langsung ke darah D. hormon melalui suatu saluran E. steroid langsung ke darah 5. Sistem respiratori penting dalam perolehan oksigen bagi jaringan dan: A. mengatur pH darah dengan mengendalikan berapa banyak oksigen yang terlarut di dalam plasma B. mengatur pH darah dengan mengendalikan berapa karbondioksida yang terlarut di dalam plasma C. mengatur tekanan darah dengan melepaskan hormon dari paru-paru yang mengendalikan laju detak jantung D. berperan dalam mengatur kadar kolesterol di dalam darah E. mengambil beberapa nutrien, seperti vitamin K, dari atmosfer saat persediaan terbatas 6. Manakah dari pernyataan berikut ini yang merupakan karakteristik dari arteri? A. Mengandung katup yang mencegah aliran balik arah B. Membawa darah menjauhi jantung C. Darah dipertahankan bergerak oleh kontraksi otot volunter D. Merupakan pembuluh yang berdinding tipis E. Selalu membawa darah yang kaya akan oksigen 9 03-OSK-2011 Copyright © 2011 by Kemdiknas Hak Cipta dilindungi oleh Undang-undang
  • 11. Nama: ......................................................................................................... Asal SMA/Kelas: ........................................................................................ Seleksi Calon Peserta IBO 2012 Tingkat Kabupaten/Kota (Kode:03) 7. Darah mengalir dari jantung ke paru-paru pada arteri pulmonaris dan kembali dari paru-paru ke jantung pada vena pulmonaris. Darah di arteri pulmonaris: A. memiliki kandungan O2 lebih tinggi dan kandungan CO2 lebih rendah dari darah yang berada di vena pulmonaris B. memiliki kandungan O2 dan CO2 lebih tinggi daripada darah yang berada di vena pulmonaris C. memiliki kandungan O2 lebih rendah dan kandungan CO2 lebih tinggi daripada darah yang berada di vena pulmonaris D. memiliki kandungan O2 dan CO2 lebih rendah daripada darah yang berada di vena pulmonaris E. memiliki kandungan O2 lebih tinggi tetapi kandungan CO2 sama dengan darah yang berada di vena pulmonaris 8. Pernyataan berikut yang TIDAK tepat mengenai pundi-pundi udara pada burung adalah: A. Berperan dalam menampung udara inspirasi atau pun ekspirasi B. Mensuplai udara kaya O2 ke paru-paru pada saat ekspirasi C. Bersama-sama dengan paru-paru melakukan pertukaran gas dengan darah D. Mengalami perubahan ukuran selama proses respirasi berlangsung E. Menampung udara dari paru-paru saat inspirasi 9. Tabel berikut adalah data mengenai kecepatan dan tekanan darah selama mengalir di sepanjang pembuluh darah: I II III IV V Tekanan darah (mm Hg) 100 22 60 2 10 Kecepatan aliran darah (cm/s) 48 0,5 15 25 4 Berdasarkan keterangan diatas maka urutan yang tepat untuk menunjukan perubahan tekanan dan kecepatan aliran darah selama mengalir dari jantung dan kembali ke jantung adalah: A. ventrikel-I-III-II-V-IV-atrium B. ventrikel-IV-V-II-III-I-atrium C. ventrikel-II-V-III-IV-I-atrium D. ventrikel-I-IV-III-V-II-atrium E. ventrikel-I-II-III-IV-V-atrium 10. Dinding arteri lebih tebal dibandingkan dinding vena. Manakah alasan yang tepat untuk mendukung struktur arteri tersebut? A. Mengalirkan darah dari jantung B. Menahan darah untuk kembali C. Mengandung oksigen dari paru-paru D. Membawa darah lebih banyak E. Jawaban A dan B benar 10 03-OSK-2011 Copyright © 2011 by Kemdiknas Hak Cipta dilindungi oleh Undang-undang
  • 12. Nama: ......................................................................................................... Asal SMA/Kelas: ........................................................................................ Seleksi Calon Peserta IBO 2012 Tingkat Kabupaten/Kota (Kode:03) 11. Apakah fungsi katup pada vena? A. Mencegah pembalikan aliran darah B. Mencegah darah C. Mencegah terbentuknya plak D. Memperluas permukaan vena E. Meningkatkan tekanan darah 12. Pertukaran gas terjadi di arteri sehingga jaringan epitel yang membatasi alveoli memiliki struktur: A. selapis kubus B. selapis pipih C. tidak ada sel epitel D. sel endotel E. silindris berlapis banyak 13. Orang yang memiliki sel darah merah sabit (sickle cell) akan memiliki masalah serius di dataran tinggi karena: A. jumlah sel darah merah B. jumlah hemoglobin di dalam darah C. konsentrasi oksigen D. tekanan udara parsial E. suhu udara 14. Berikut ini adalah molekul-molekul yang terdapat di dalam plasma darah: I. Urea II. Air III. Glukosa IV. Protein V. Antibodi VI. Ion hidrogen Berapa macam dari molekul di atas yang diekskresikan atau direabsorbsi di ginjal? A. 3 B. 2 C. 4 D. 5 E. 6 15. Seorang meneliti tiga jenis rodensia yang berkerabat dekat. Yang menjadi objek kajian dalam penelitian tersebut adalah anatomi perbandingan dari struktur ginjal ketiga jenis rodensia. Hasil penelitian menunjukan bahwa ketiga jenis rodensia memiliki panjang lengkung Henle yang berbeda. Berdasarkan penemuan tersebut, hipotesis yang mungkin dibangun adalah: A. ketiga rodensia memiliki laju filtrasi ginjal yang berbeda-beda B. ketiga rodensia memiliki kepekatan urin yang berbeda-beda C. ketiga rodensia menghasilkan urin dengan pH yang berbeda-beda D. ketiga rodensia memproduksi urin dengan volume yang sama E. ketiga rodensia hidup pada habitat yang sama 11 03-OSK-2011 Copyright © 2011 by Kemdiknas Hak Cipta dilindungi oleh Undang-undang
  • 13. Nama: ......................................................................................................... Asal SMA/Kelas: ........................................................................................ Seleksi Calon Peserta IBO 2012 Tingkat Kabupaten/Kota (Kode:03) 16. Sel-sel dari kelenjar berikut mensekresikan hormon yang berperan meningkatkan kadar gula darah dan mempercepat kerja jantung serta pernafasan ketika tubuh sedang dalam kondisi tercekam. Kelenjar yang dimaksud adalah: A. pituitari B. adrenal C. pankreas D. tiroid E. hipothalamus 17. Perhatikan gambar berikut ini: Seorang dokter dapat memeriksa kondisi kehidupan dari pasiennya dengan memberikan sorotan lampu senter ke arah mata. Pada orang yang masih hidup, pemberian cahaya akan direspon melalui perubahan diameter bagian (......). Akan tetapi, pada orang mati bagian tersebut tidak akan merespon terhadap cahaya senter. A. A B. B C. C D. D E. E 18. Berdasarkan bentuknya, karakter yang tepat untuk mewakili sel saraf berikut adalah: 12 03-OSK-2011 Copyright © 2011 by Kemdiknas Hak Cipta dilindungi oleh Undang-undang
  • 14. Nama: ......................................................................................................... Asal SMA/Kelas: ........................................................................................ A. B. C. D. E. Seleksi Calon Peserta IBO 2012 Tingkat Kabupaten/Kota (Kode:03) berperan dalam fungsi sensoris dan ditemukan di sistem saraf pusat berperan dalam fungsi motoris dan ditemukan di sistem saraf tepi berperan dalam fungsi sensoris dan ditemukan dalam sistem saraf tepi berperan dalam fungsi motoris dan ditemukan dalam sistem saraf pusat berperan dalam fungsi integrasi dan ditemukan di sistem saraf pusat 19. Sistem saraf pada manusia di bagi menjadi sistem saraf pusat dan sistem saraf tepi (perifer). Manakah dari pernyataan berikut ini yang tepat mengenai sistem saraf tepi? A. Sistem saraf tepi terletak di otak dan susum tulang belakang B. Sistem saraf tepi mengandung sel-sel saraf yang merupakan perpanjangan dari sistem saraf pusat C. Sistem saraf tepi mengandung interneuron dari sistem saraf pusat D. Sistem saraf tepi merupakan sistem saraf pusat yang menjadi bagian dari susum lanjutan E. Sistem saraf tepi merupakan pusat pengaturan sistem reflek 20. Salah satu indikasi terjadinya infeksi di dalam tubuh adalah demam. Demam timbul karena produksi pirogen oleh sel darah putih pada saat infeksi terjadi. Pirogen mempengaruhi ..... di otak untuk meningkatkan nilai acuan suhu tubuh dari 37 ke suhu yang lebih tinggi. Usaha tubuh untuk meningkatkan suhu tubuh inilah yang menimbulkan sensasi kedinginan pada saat demam. Pilihlah jawaban yang tepat untuk melengkapi keterangan di atas. A. Hipofisa B. Hipotalamus C. Epifisa D. Sistem limbik E. Korteks otak besar 21. Ketika seseorang yang alergi seruk sari terpapar dengan serbuk sari, matanya akan berair dan timbul bintik-bintik merah di permukaan kulitnya. Peristiwa ini terjadi akibat pelepasan: A. antigen oleh sel darah merah B. enzim oleh platelet C. histamin oleh sel-sel mast D. hormon oleh kelenjar pituitari E. mukus oleh sel-sel eksokrin 22. Perhatikan diagram perkembangan berikut ini. Istilah yang tepat untuk menggambarkan A dan B adalah: A. A: Ovulasi, B : Diferensiasi B. A: Meiosis, B : Organogenesis 13 03-OSK-2011 Copyright © 2011 by Kemdiknas Hak Cipta dilindungi oleh Undang-undang
  • 15. Nama: ......................................................................................................... Asal SMA/Kelas: ........................................................................................ Seleksi Calon Peserta IBO 2012 Tingkat Kabupaten/Kota (Kode:03) C. A: Proses pembelahan (cleavage), B: Diferensiasi D. A: Germinasi, B: Diferensiasi E. A: Morulasi, B: Gastrulasi 23. Perbedaan mendasar antara tulang dan rawan adalah bahwa rawan: A. merupakan salah satu tipe jaringan ikat B. tidak memiliki pembuluh darah dan saraf C. mensekresi matriks padat D. tersusun atas kolagen dan garam E. merupakan bagian dari sistem rangka 24. Komponen berikut ini yang konsentrasinya meningkat di sitoplasma pada saat otot berkontraksi adalah: A. ATP B. Ca2+ C. Mg2+ D. Aktin E. Myosin 25. Berikut ini adalah urutan kejadian yang terjadi pada otot lurik. Molekul apakah yang diwakili oleh X? A. hemoglobin B. glikogen C. asam amino D. alkohol E. asam laktat 14 03-OSK-2011 Copyright © 2011 by Kemdiknas Hak Cipta dilindungi oleh Undang-undang
  • 16. Nama: ......................................................................................................... Asal SMA/Kelas: ........................................................................................ Seleksi Calon Peserta IBO 2012 Tingkat Kabupaten/Kota (Kode:03) ETOLOGI (Nilai 5) 1. Penjelasan yang paling baik untuk menjelaskan perilaku manusia yang tersenyum pada saat menyapa teman walaupun mereka berasal dari komunitas yang berbeda adalah: A. Mereka memiliki beberapa kultur yang umum B. Mereka mengingat wajah yang tersenyum pada saat mereka masih bayi C. Mereka telah mempelajari bahwa senyum tidak memicu perilaku agresif D. Senyum merupakan suatu pola perilaku yang diturunkan secara genetis E. Senyum adalah suatu perilaku yang dipelajari 2. Merkat merupakan hewan sosial yang hidup di daearah savana di Afrika. Hewan ini hidup pada sistem terowongan di dalam tanah yang memungkinkan mereka untuk melarikan diri dari pemangsa. Untuk mengetahui kedatangan pemangsa, koloni Merkat mengandalkan kepada sekelompok kecil individu yang menjadi pengintai di luar sistem koloni. Pada saat pemangsa datang, pengintai ini mengeluarkan suara peringatan sampai seluruh anggota koloni aman di dalam sistem terowongan. Perilaku ini merupakan salah satu contoh dari: A. social learning B. conditional learning C. association learning D. alturism E. imprinting 3. Karl von Firsch merupakan salah satu orang penting dalam dunia penelitian perilaku hewan. Beliau berhasil menerjemahkan bahasa lebah pekerja yang memberikan informasi posisi sumber makanan. Bahasa lebah tersebut dikenal dengan istilah round dance dan waggle dance. Hasil penelitian Karl von Firsch tersebut menunjukkan peranan (.....) dalam menentukan perilaku hewan. A. belajar B. tanda dan stimulus C. hadiah dan hukuman D. pengulangan stimulus E. hubungan sosial 4. Pada hewan, perilaku ritual dengan sedikit risiko kecelakaan serius atau kematian pada pelakunya pada populasi adalah: A. hirarki dominansi yang stabil B. altruisme biologis C. radiasi adaptif D. perilaku insting E. habituasi 5. Burung hitam Eropa terkenal dengan kemampuannya untuk melakukan migrasi yang terkondisi. Hal ini dipicu ketika: A. persaingan antara setiap individu mulai semakin parah. B. kondisi musim memungkinkan mereka untuk melakukan navigasi berdasarkan kepada pola-pola yang terdapat di langit. C. keuntungan dari mengkolonisasi habitat baru melebihi bahaya yang didapatkan dalam perjalanan menuju ke lokasi tersebut D. jumlah dari individu-individu yang dominan secara sosial menurun tajam. E. semua pernyataan di atas salah 15 03-OSK-2011 Copyright © 2011 by Kemdiknas Hak Cipta dilindungi oleh Undang-undang
  • 17. Nama: ......................................................................................................... Asal SMA/Kelas: ........................................................................................ Seleksi Calon Peserta IBO 2012 Tingkat Kabupaten/Kota (Kode:03) GENETIKA DAN EVOLUSI (Nilai 20) 1. Hukum Mendel pertama menyatakan bahwa unit-unit penurunan sifat yang mengontrol fenotip berbeda dapat diturunkan secara terpisah. Manakah dari pernyataan berikut yang paling sesuai dengan bunyi hukum tersebut? A. Mutasi merupakan kejadian acak yang mempengaruhi setiap lokus secara terpisah B. Saat fertilisasi, sperma mana yang akan membuahi sel telur merupakan peristiwa acak C. Saat meiosis I, kromosom berpasangan dengan homolog D. Saat meiosis I, pemisahan satu pasangan kromosom tidak mempengaruhi pemisahan pasangan kromosom lainnya E. Saat mitosis, tidak terdapat perbedaan antara DNA induk dengan DNA hasil replikasi 2. Pada jenis ayam tertentu, gen untuk bulu hitam dan bulu putih bersifat kodominan. Ayam jenis ini akam memiliki: A. tiga tipe fenotip untuk warna bulu B. dua fenotip untuk warna bulu C. hanya dua genotip untuk warna bulu D. hanya memiliki bulu bewarna hitam atau putih saja E. tiga genotip untuk bulu warna hitam 3. Seorang peneliti melakukan persilangan diantara dua tikus yang keduanya memiliki rambut hitam. Rambut hitam dominan terhadap putih. Sebanyak 75% keturunan berambut hitam dan sisanya berambut putih. Peneliti dapat berasumsi bahwa genotip parental paling mungkin adalah: A. BB x BB B. BB x Bb C. BB x bb D. Bb x Bb E. bb x bb 4. Suatu sel abnormal memiliki pasangan kromosom yang dinyatakan dengan A1/A2 B1/B1 C1/C2 D1, dimana kromosom D tidak memiliki pasangan homolog. Berdasarkan tipe kromosom penyusunnya, berapa jenis sel gamet yang anda harap dapat dihasilkan dari turunan sel tersebut? A. 8 B. 7 C. 6 D. 4 E. 2 5. Saat ini banyak sekali orang takut terkena penyakit kanker jika memiliki orang tua yang terserang kanker. Salah satu penyebab timbulnya kanker adalah akibat adanya mutasi pada gen-gen yang mengatur siklus sel. Ketakutan dari orang-orang tersebut beralasan apabila gengen yang termutasi ditemukan pada (.....) dari orang tuanya. A. sel hati B. sel pankreas C. sel saraf D. sel-sel yang berada di dalam tubulus seminiferus E. sel-sel penyusun jaringan ikat di ovarium 16 03-OSK-2011 Copyright © 2011 by Kemdiknas Hak Cipta dilindungi oleh Undang-undang
  • 18. Nama: ......................................................................................................... Asal SMA/Kelas: ........................................................................................ 6. Seleksi Calon Peserta IBO 2012 Tingkat Kabupaten/Kota (Kode:03) Sifat tinggi batang dari tanaman X diatur secara poligenik oleh dua alel (A dan B) yang terdapat pada kromosom berbeda. Tanaman dengan genotip AABB memiliki tinggi 18 cm, sedangkan tanaman dengan genotipe aabb memiliki tinggi 10 cm. Jika persilangan antara dua tanaman AaBb x AaBb menghasilkan 160 keturunan. Berapa banyak dari keturunan tersebut diharapkan memiliki tinggi 16 cm? A. 150 B. 90 C. 60 D. 30 E. 10 Untuk pertanyaan No.7-8: Perhatikan pohon silsilah berikut ini. Warna abu-abu menunjukkan individu yang menderita suatu penyakit. I 1 2 2 3 II 1 4 5 III 1 7. 2 3 4 IV V 1 2 V Berdasarkan pohon silslah di atas, bagaimanakah pola penurunan penyakit ini? A. B. C. D. E. Autosomal resesif Autosomal dominan Terpaut X dominan Terpaut X resesif Terpaut Y 8. Jika IV-1 kawin dengan seorang laki-laki yang tidak menderita penyakit tersebut, berapa peluang ia akan melahirkan anak laki-laki yang sehat? A. 0 % B. 25 % C. 50 % D. 75 % E. 100 % 9. Pada Drosophila melanogaster, alel dominan (w+) mengatur munculnya sifat mata merah, sedangkan alel resesif (w) yang merupakan alel mutan, mengatur munculnya sifat mata putih. Alel ini terpaut kromosom X. Jika perkawinan antar dua Drosophila menghasilkan keturunan ½ mata putih dan ½ mata merah, maka genotip dari kedua Drosophila tersebut adalah: A. Xw+Xw+ vs. XwY B. XwXw+ vs. Xw+Y C. Xw+Xw vs. XwY D. XwXw vs. Xw+Y E. XwXw vs. XwY 17 03-OSK-2011 Copyright © 2011 by Kemdiknas Hak Cipta dilindungi oleh Undang-undang
  • 19. Nama: ......................................................................................................... Asal SMA/Kelas: ........................................................................................ Seleksi Calon Peserta IBO 2012 Tingkat Kabupaten/Kota (Kode:03) 10. Sebuah mutasi nonsense terjadi di tengah urutan coding gen M. Apakah yang anda harapkan terjadi pada ukuran produk transkripsi (panjang basa) dan translasi (Dalton) dari gen M tersebut dibanding produk akhir dari gen M normal? A. Produk transkripsi memiliki ukuran yang lebih kecil, sedangkan produk translasi tidak berubah B. Produk transkripsi tidak berubah, sedangkan produk translasi memiliki ukuran yang lebih kecil C. Produk transkripsi dan translasi memiliki ukuran yang lebih kecil D. Produk transkripsi dan translasi tidak berubah E. Tidak dihasilkan produk transkripsi dan translasi 11. Suatu populasi yang berada dalam kesetimbangan Hardy-Weinberg memiliki dua alel untuk suatu lokus, A dan a. Jika frekuensi A sebesar 0,7; berapa frekuensi individu heterozigot pada populasi tersebut? A. 0,09 B. 0,42 C. 0,49 D. 0,51 E. 0,91 12. Suatu populasi berada dalam kesetimbangan genetik ketika frekuensi alel-alel dari suatu gen pada populasi tersebut tetap sama dari satu generasi ke generasi berikutnya. Kesetimbangan genetik akan lebih mudah tercapai apabila: A. populasi berukuran kecil sehingga lebih rentan terhadap genetic drift B. populasi memperoleh mutasi yang menguntugkan C. populasi tidak mengalami imigrasi dan emigrasi D. terjadi perkawinan antar kerabat dekat E. seleksi alam bekerja terhadap fenotip tertentu Untuk pertanyaan No.13-14: Perhatikan gambar berikut ini. 13. Manakah dari kurva di atas yang paling baik menggambarkan konsep seleksi penstabilan (stabilizing selection) atau seleksi yang lebih menyukai varian intermediet daripada individuindividu dengan fenotip ekstrim? A. Hanya grafik I B. Hanya grafik II C. Grafik I dan grafik II D. Hanya grafik III E. Grafik II dan grafik III 18 03-OSK-2011 Copyright © 2011 by Kemdiknas Hak Cipta dilindungi oleh Undang-undang
  • 20. Nama: ......................................................................................................... Asal SMA/Kelas: ........................................................................................ Seleksi Calon Peserta IBO 2012 Tingkat Kabupaten/Kota (Kode:03) 14. Manakah dari kurva di atas yang paling baik menggambarkan konsep seleksi diversifikasi atau tipe seleksi yang lebih menyukai fenotip ekstrim daripada fenotip intermediet? A. Hanya grafik I B. Hanya grafik II C. Grafik I dan grafik II D. Hanya grafik III E. Grafik II dan grafik III 15. Sebelum kedatangan bangsa Eropa, Amerika Utara merupakan habitat bagi jutaan ayam padang rumput (Tympanuchus cupido). Akibat perburuan liar dan perusakan habitat, ukuran populasi mereka menurun drastis hingga hampir mencapai kepunahan. Keanekaragaman genetik ayam padang rumput ikut berkurang sehingga kesuksesan reproduktif mereka menurun. Mekanisme evolusi apakah yang terjadi pada populasi tersebut? A. Aliran gen (gene flow) B. Genetic drift C. Mutasi D. Perkawinan tidak acak E. Seleksi buatan 16. Walaupun anjing laut dan penguin sama-sama memiliki tubuh ramping yang cocok untuk berenang dan dilapisi oleh lemak, kedua hewan ini tidak berkerabat dekat. Kesamaan ini dihasilkan oleh: A. koevolusi B. evolusi konvergen C. radiasi adaptif D. seleksi terarah E. spesiasi 17. Pada kondisi tertentu, suatu populasi dapat berkembang menjadi dua atau lebih spesies tanpa keberadaan pemisah geografis antara lain dengan perbedaan pada perilaku. Perilaku yang terdapat di bawah ini dapat memicu terjadinya spesiasi, KECUALI: A. seleksi dari buah yang menjadi inang secara spesifik oleh lalat buah B. perilaku kawin berbeda yang dikembangkan oleh burung layang-layang pada habitat yang berbeda C. pemilihan waktu untuk berkembang biak oleh kupu-kupu D. pelepasan gamet ke lautan oleh invertebrate laut sebagai bentuk respons terhadap senyawa kimia pada air E. pemilihan tipe mangsa oleh elang yang memiliki kisaran mangsa yang luas 18. Dua populasi bekicot terisolasi secara baik dalam jangka waktu panjang. Berdasarkan kepada konsep spesies, manakah dari pernyataan berikut yang menunjukkan bahwa kedua populasi ini telah menjadi dua spesies yang berbeda? A. Kedua populasi memiliki sekurang-kurangnya lima sifat morfologis yang berbeda B. Kedua populasi memiliki urutan DNA yang berbeda C. Kedua populasi memiliki perbedaan kandungan protein D. Kedua populasi memiliki respon yang berbeda pada saat terkena pestisida dengan dosis yang sama E. Hibrid steril akan dihasilkan ketika anggota dari kedua populasi tersebut disilangkan 19 03-OSK-2011 Copyright © 2011 by Kemdiknas Hak Cipta dilindungi oleh Undang-undang
  • 21. Nama: ......................................................................................................... Asal SMA/Kelas: ........................................................................................ Seleksi Calon Peserta IBO 2012 Tingkat Kabupaten/Kota (Kode:03) 19. Bipedalisme diyakini telah berevolusi pada garis nenek moyang manusia karena pergerakan bipedal: A. lebih efisien daripada pergerakan quadrupedal, karena dapat memberi kebebasan bagi anggota gerak depan untuk memanipulasi objek B. lebih efisien daripada pergerakan quadrupedal karena energi yang diperlukan lebih sedikit C. kurang efisien daripada gerakan quadrupedal, tetapi dapat memberi kebebasan bagi anggota gerak depan untuk memanipulasi objek D. kurang efisien daripada gerakan quadrupedal, tetapi hewan bipedal dapat berlari lebih kencang E. kurang efisien daripada gerakan quadrupedal, tetapi seleksi alam tidak bekerja untuk memperbaiki efisiensi 20. Pada awal kehidupan, organisme hanya ditemukan di lautan. Namun dengan berkembangnya kerak benua, dan terdapatnya daratan yang luas, makhluk hidup mulai menginvasi daratan. Begitu pula dengan tumbuhan. Pada tumbuhan, organ yang terbentuk sebelum menginvasi daratan adalah: A. dinding sel B. kutikula C. spora D. akar E. stomata 20 03-OSK-2011 Copyright © 2011 by Kemdiknas Hak Cipta dilindungi oleh Undang-undang
  • 22. Nama: ......................................................................................................... Asal SMA/Kelas: ........................................................................................ Seleksi Calon Peserta IBO 2012 Tingkat Kabupaten/Kota (Kode:03) EKOLOGI (Nilai 10) Untuk pertanyaan No.1-2: Perhatikan grafik pertumbuhan penduduk dari lima negara berikut ini. 1. Negara mana yang mengalami petumbuhan penduduk paling pesat? A. (A) B. (B) C. (C) D. (D) E. (E) 2. Negara mana yang memiliki petumbuhan penduduk mendekati nol? A. (A) B. (B) C. (C) D. (D) E. (E) 3. Mimikri adalah salah satu strategi yang dihasilkan sebagai proses dari seleksi alami yang berperan besar dalam meningkatkan kelulushidupan dari suatu organisme. Manakah dari pernyataan berikut ini yang tergolong sebagai bentuk dari mimikri Batesian? A. Lipan yang memiliki senyawa racun bagi katak dan selalu dihindari oleh katak setelah katak terkena racun lipan tersebut pada saat proses pemangsaan sebelumnya B. Ngengat yang memiliki gambar seperti “mata palsu” pada bagian sayap belakang yang berperan untuk membingungkan musuhnya C. Anak camar yang memiliki warna serupa dengan lingkungan tempat dia hidup D. Anak camar yang dapat memperoleh makanan dari burung selain induknya dengan mengetuk titik yang tepat pada paruh dari burung tersebut E. Ngengat yang memiliki warna serupa dengan lebah penyengat 4. Energi yang tersimpan dalam organisme pada suatu tingkatan trofik tertentu adalah: A. kira-kira 10% dari energi pada tingkatan trofik di bawahnya B. kira-kira 10% dari energi pada tingkatan trofik di atasnya C. kira-kira 50% dari energi pada tingkatan trofik di bawahnya D. kira-kira 50% dari energi pada tingkatan trofik di atasnya E. 100% dari energi pada tingkatan trofik di bawahnya 21 03-OSK-2011 Copyright © 2011 by Kemdiknas Hak Cipta dilindungi oleh Undang-undang
  • 23. Nama: ......................................................................................................... Asal SMA/Kelas: ........................................................................................ Seleksi Calon Peserta IBO 2012 Tingkat Kabupaten/Kota (Kode:03) 5. Pada tingkatan konsumen primer, proses utama yang mengurangi energi sehingga tidak menjadi biomassa yang dapat dimanfaatkan pada tingkatan di atasnya adalah: A. fotosintesis B. ekskresi C. pertumbuhan D. perkembangan E. respirasi 6. Diantara kelompok organisme berikut, manakah yang dapat berperan sebagai konsumen sekunder maupun tersier? A. Dekomposer B. Herbivora C. Detritivora D. Autotrof E. Karnivora 7. Kelompok tumbuhan klimaks pada proses suksesi sekunder terestrial umumnya berasal dari kelompok: a. rumput b. herba c. perdu dan semak d. pohon berkayu keras e. pohon konifer 8. Manakah dari pernyataan berikut ini yang tepat mengenai siklus hidrologi? A. Karena air diputarkan dalam siklus, maka manusia tidak akan pernah kehabisan air tawar untuk dimanfaatkan. B. Sebagian air mengalami evaporasi dari daratan dan dari tumbuhan C. Semua molekul air yang dievaporasi dari lautan akan jatuh sebagai hujan di daratan D. Seluruh air yang turun sebagai hujan akan mengalami gaya gravitasi sehingga akan terbawa kembali ke laut E. Setelah terserap ke dalam tanah, air menjadi aman dari eksploitasi dan pencemaran oleh manusia 9. Tumbuhan mengambil nitrogen dari dalam tanah dalam bentuk molekul: A. N2 B. NO C. NO2 D. NO3E. NO4+ 10. Energi dalam suatu ekosistem mengalir tetapi tidak diputarkan (tidak membentuk siklus) karena: A. energi hilang (musnah) setelah terpakai B. energi tersebar merata pada semua organisme C. energi dikonversi menjadi berbagai bentuk energi yang bermanfaat D. energi meningkat sejalan dengan naiknya tingkatan trofik E. energi tidak dapat digunakan jika sudah terkonversi menjadi panas 22 03-OSK-2011 Copyright © 2011 by Kemdiknas Hak Cipta dilindungi oleh Undang-undang
  • 24. Nama: ......................................................................................................... Asal SMA/Kelas: ........................................................................................ Seleksi Calon Peserta IBO 2012 Tingkat Kabupaten/Kota (Kode:03) BIOSISTEMATIK (Nilai 5) 1. Berikut ini manakah yang merujuk pada taksa tingkat famili dari organisme? A. Solanum B. Pandanales C. Felidae D. Ipomoea E. Glycine 2. Kelompok tumbuhan yang memiliki biji yang tertutup oleh daun buah (perikarp) adalah: A. Monokotil B. Dikotil C. Angiospermae D. Gymnospermae E. Spermatophyta 3. Salah satu argumentasi yang menyebabkan jamur (fungi) di kelompokkan ke dalam satu kerajaan khusus adalah: I. Jamur tidak memiliki klorofil II. Jamur dapat berkembang biak secara generatif dan vegetatif III. Jamur merupakan organisme heterotrof IV. Jamur berkembang biak dengan spora V. Bagian-bagian jamur tidak dapat dibedakan antara daun, batang dan akar Manakah dari kombinasi pernyataan di atas yang benar? A. I, II, III B. II, III, IV C. III, IV, V D. I, III E. I, V 4. Dalam daur hidup katak, stadium yang paling muda ditandai dengan: A. terbentuknya kaki depan B. terbentuknya kaki belakang C. menghilangnya ekor D. bernafas dengan insang luar E. bernafas dengan insang dalam 5. Organisme berikut ini termasuk kelas Mammalia yang hidup di perairan, KECUALI: A. Pesut B. Paus C. Kuda Laut D. Dugong E. Lumba-lumba 23 03-OSK-2011 Copyright © 2011 by Kemdiknas Hak Cipta dilindungi oleh Undang-undang